.
.
Math Central - mathcentral.uregina.ca
Quandaries & Queries
Q & Q
. .
topic card  

Topic:

fraction

list of
topics
. .
start over

507 items are filed under this topic.
 
Page
1/1
How to convert fractions to percentages 2007-07-05
From Anita:
How do I turn 1 3/20 to a percent?
Answered by Stephen La Rocque and Melanie Tyrer.
The next rational number 2022-01-14
From Sophia:
what comes after 3 1/2?
Answered by Penny Nom.
Simplify 2021-02-05
From Leslie:
To simplify improper fractions, do you convert it to a mixed number or leave as an improper fraction?

My daughter had a quiz in which she was supposed to simplify her answers. So, for example, on one question the answer she got was 11/10 and on another question she came up with 17/12. The teacher marked these as incorrect because they were mixed numbers. Her teacher wrote on her paper, "**name**, I even gave this back to you and told you to go through and simplify your answers!" We (her father and I) feel that the teacher wanted her to convert her answers, not simplify them but I haven't been able to find a definitive answer online. So, does simplifying improper fractions mean converting them to mixed numbers? Thank you for your help!


Answered by Penny Nom.
Equivalent fractions 2020-10-31
From Lexi:
Which of the following fractions is equivalent to -84 / -90?
A. -14 / 15
B. -42 / 45
C. 14 / 15
D. 42 / 45

Answered by Penny Nom.
Two equations with fractions 2020-10-22
From mia:
I am supposed to look for x and y. my teacher didn't explain very well.
please help : ((
2x - y = 5
x/4 + y/3= 2

Answered by Penny Nom.
A word problem with fractions 2020-06-21
From Sthantsa:
I'm stuck with this question, I don't know where to begin because i really don't Know where to start.

Jessie and Thulani each has a sum of money. Jessie’s amount is 2/5 that of Thulani’s. If Thulani were to give Jessie R198, then his remaining amount will be 6/8 that of Jessie’s amount.
How much does Jessie have originally??

Answered by Penny Nom.
390 as a percentage of 800 2020-06-18
From Yug:
What percentage is 390 grams of 800 grams?
Answered by Penny Nom.
10 of 24 hours as a percentage 2020-05-27
From Larzy:
what is 10 of 24 hours in percentages
Answered by Harley Weston.
A question about fractions 2020-01-26
From Awotile:
A fraction whose denominator is more than the numerator is double when the numerator is increased by 6 and the denominator is increased by 5 find the original fraction?
Answered by Penny Nom.
3/8 and 0.375 2019-12-05
From Zamyiah:
how does 3/8 turn into 0.375 i just don't understand it the question is hard. please answer in time thank you for your time.
Answered by Penny Nom.
Simultaneous equations 2019-10-16
From deepak:
This is too complicated please help me
8/x - 10/y = 1 and x+y=9

Answered by Penny Nom.
Fractions and negative powers 2019-09-22
From Will:
-2/3 to the -6 power
Answered by Penny Nom.
400 in 7.5 billion 2019-08-28
From Emma:
What is a better way to express 400 in 7.5 billion? My daughter has a rare condition, I want to know the answer to say “she is 1 in a ...”
Answered by Penny Nom.
150 litres has been drawn from a tank 2019-06-12
From Eke:
When 150 litres has been drawn from a tank, it is 3/8 full, how many litres will the tank hold?
Answered by Penny Nom.
The exponential form of sqrt(13) 2019-02-10
From isaebl:
what is the exponential form of sqrt(13)
Answered by Penny Nom.
Fractions with roots 2019-02-06
From DEBMALYA:
5/√2+√3 –1/√2–√3
Answered by Penny Nom.
A fraction of a cent 2019-02-01
From michael:
$ 300 divided by 1,000,000 is 00.0003 ... so, is that three hundredths of one ($ 00.0003) cent or what ... thanks
Answered by Penny Nom.
One quarter of a quarter cake 2019-01-30
From Tre:
1/4 of a birthday cake was left over from a party. The next day, it is shared among 4 people. How big a piece of the original cake did each person get?
Answered by Penny Nom.
A puzzling equation 2018-12-04
From Manny:
Hello,

Can you explain why this is wrong:
-1 = (-1)^3 = (-1)^{6/2} = ((-1)^6)^{1/2} = (1)^{1/2} = 1 ??

Thank you

Answered by Penny Nom.
Ordering fractions 2018-11-25
From Mae:
order these from least to greatest:

19/6, 3 5/14, 83/21

Answered by Penny Nom.
2/5 as much time 2018-11-16
From Izzy:
If Patricia spent 3/8 of her time cleaning her room. She spent 2/5 as much time washing the dishes as she did cleaning her room. What fraction of time did Patricia spend Washing dishes? And it wants us to simplify our answer
Answered by Penny Nom.
-4 + 3/2 2018-11-15
From Chloe:
so I have this paper and I'm confused, -4 + 3/2=???? so I originally got -1/2 but I looked at Calculator papa and he says -5/2 and then I looked at google and all I got was -2.5 and i am so confused and lost.
Answered by Penny Nom.
Dividing by a fraction 2018-11-02
From Fadely:
If 8 divided by 4/2 can be expressed as 8 X 2/4, how can 8 X 2/4 be converted back to 8 divided by 4/2?
Answered by Penny Nom.
A cube in a cylinder 2018-08-13
From Geli:
A cylinder with a radius r and height 2r+4 contains a cube with edge length r√2. What fraction of cylinder volume is taken up by the cube
Answered by Penny Nom.
Write these fractions from greatest to the least 2018-05-18
From angela:
math fraction greater to the least

1\5 ,1\2 or 1\4

Answered by Penny Nom.
Converting decimals to common fractions 2018-05-08
From Ruth:
what does the 0 represent in .018 versus the decimal .180? As in, changing these to fractions? Thank you.
Answered by Penny Nom.
A fraction 2018-05-02
From Adwin:
The denominator of a rational number is greater than its numerator by 8. If the numerator is increased by 17 the number obtained is 3 by 2. Find the number.
Answered by Penny Nom.
Walking a fraction of a mile 2018-03-20
From Jamie:
Who walked a fraction of a mile that is closer to neither 0 nor 1 ? Explain
Avery 1/6
me.Nunez 5/6
Ms. Chang 1/3
Mr. O’Leary 4/8
Miss Lee 4/6

Answered by penny Nom.
Which movie lasts longer? 2018-03-14
From YKJAJA:
Which movie lasts longer than the other?
A) “space adventure” lasts for 1 2/3 hour
B) “animal world” lasts for 15/12 hour

Answered by Penny Nom.
The decimal form of a fraction 2018-01-12
From Tom:
Prove that the decimal representation of the quotient of 2 integers must repeat (if it does not terminate).
Answered by Penny Nom.
A fraction of a mile 2018-01-06
From Sierra:
There are 5280 feet in a mile. what fraction of a mile is represented by 660 feet?
Answered by Penny Nom.
A puzzling sequence 2017-12-19
From Alan:
My son has the following sequence to work out the formula for the nth term (and fing 5th and 6th terms) and I have tried a variety of ways but can't help!

1/2, 5/6, 1, 11/10

I have converted all to 15/30, 25/30, 30/30, 33/30 and so think the next two terms are 37/30 and 45/30 but I have no idea how to prove or if I am right!

Any help appreciated.

Answered by Penny Nom.
Splitting twice 2017-11-11
From Terry:
4 people are splitting 1/3 of something. 1 is to receive twice as much as the other 3. what it the fractional interest for each?
Answered by Penny Nom.
Adding fractions 2017-10-27
From Sandy:
virinchi distributed 4/7 of cake to his friends 1/3 of it to his family members which part of cake is remaining?
Answered by Penny Nom.
Changing a mixed number to a decimal 2017-09-23
From karla:
to change an improper fraction into a decimal 5 3/4 equals
Answered by Penny Nom.
Simultaneous equations with fractions 2017-09-09
From Farah:
Hi, my name is Farah. I hope you can help me with this question . X/2 + g/5= 3 and 2g - f = 10
Answered by Penny Nom.
((a-5)/a)/(4/a) 2017-07-18
From Michael:
Find the Quotient:

((a-5)/a)/(4/a)

Answered by Penny Nom.
Ordering fractions 2017-05-25
From Esmeralda:
How May i order this 30%,3/100,7/20 and 0.33 from least to greatest
Answered by Penny Nom.
Simultaneous equations with fractions 2016-12-21
From Kimi:
I am stuck on this,can someone please help me????

1/2x+1/3y=11
8x+2/5y=102
Can someone please explain how to solve questions like these?? Or else I will never learn...
Thank you

Answered by Penny Nom.
The weights of two boxes 2016-11-25
From dearbhla:
One box weighs 3 3/7kg. Another box weighs 1 5/6 as much. How heavy is the second box?
Answered by Penny Nom.
Why are equivalent fractions equal? Part 2. 2016-11-22
From Anandmay:
Hi! I saw one of your answered questions:Why are equivalent fractions equal? The same question I had for why i searched and ended up entering this wonderful site. The answer was so much experimental based.I mean,for example,it was explained how 3/4 was equal to 6/8 by dividing a unit into 4 equal parts and taking four of the parts,and then dividing the same unit into 8 equal parts and then taking 6 of them.The result was that both the taken parts were equal. (That is,3 out of 4 equal parts were equal to 6 of the 8 equal parts.) But,this was very much non-generalized and experimental-based result(proof). And we all know that in mathematics,we confirm some particular thing for all the numbers iff we generalize it.

So,can you explain more clearly 'HOW' the 3 parts of the 4 equal parts and the 6 of the 8 equal parts of the unit equal(that is the same)?I don't want experimental proof.I want clear proof,and thus please give a generalized proof for all fractions and their equivalents.
Thanks!

Answered by Penny Nom.
An equation with fractions 2016-11-09
From Tina:
How do you solve x+1/3=x+2/5. X+1 divided by 3= x+2 divided by 5. What's x=
Answered by Penny Nom.
5/8 of 16 2016-08-14
From Michael:
What is the fraction of 5/8 of 16
Answered by Penny Nom.
Two equations with fractions 2016-07-22
From kanesh:
p/2+q/3=3

p/4+2q/3=3

Answered by Penny Nom.
Writing numbers in the form a/b 2016-07-17
From Edward:
How to write these numbers in form of a/b: -7, 0.175, -3.25, 37/10, -16, 1/3, 2.04
Answered by Penny Nom.
Two pieces of rope 2016-05-21
From Render:
Fred cuts a 12- inch piece of rope. Then he cuts a second piece of rope that is 3/2 longer than the first piece. Is the second piece shorter or longer than the first piece? Explain.
Answered by Penny Nom.
A pair of equations with fractions 2016-05-11
From tiya:
hello, i want to know how to solve this question.

m/6+2n/3=6
-m/10=2n/5=2

Answered by Penny Nom.
An equation with fractions 2016-03-09
From Ed:
7/10-___ +3/2=6/5
Answered by Penny Nom.
Converting a fraction to a percentage 2016-02-22
From Preitty:
How can I turn fractions into percentages when the denominator is not a number that can multiply to 100?
Answered by Penny Nom.
Exponential form 2016-01-26
From Jufe:
What is the exponential form of ³√11x²
Answered by Penny Nom.
Fractions of two quantities 2016-01-22
From Melody:
Kate ate 1/4 of her orange. Ben ate 2/4 of his banana. Did Kate and Ben eat 3/4 of theit fruit? Explain.
Answered by Penny Nom.
Solve 1/3 ( x-5 ) + 4 =1/6 ( 2 - 3 x ) +1 for x 2016-01-15
From edwin:
1/3 ( x-5 ) + 4 =1/6 ( 2 - 3 x ) +1
Answered by Penny Nom.
Converting a fraction to a percent 2015-12-22
From Sharon:
How do you turn a fraction into a percentage? I'm beyond confused with this. Even though I think of myself as a good math student doing 8th grade math at a young age, this just makes me want to scream! I get the way you turn 1/4 into 25%, but when it comes to something like 7/9 = ? I just can't figure out the answer.
Answered by Penny Nom.
Dividing by a fraction 2015-11-21
From Jennifer:

Question from Jennifer, a student:

I want to know how to get the right answer for this math problem, dividing a fraction in to a whole number , example 3/8 using the whole number 6? Thanks


Answered by Penny Nom.
Working with fractions 2015-11-12
From muhammad:
There are 1200 pupils in a school, 2/3 of them are girls, 1/4 of the boys are overweight.How many boys are overweight?
Answered by Penny Nom.
Is 22/7 bigger or smaller than 3.14? 2015-11-06
From Natalie:
Is 22/7 bigger or smaller than 3.14?
Answered by Harley Weston.
Mary, Julie and Peter when on a trip 2015-07-10
From Edward:
How do you solve and set up a problem where: Mary pays one third of the cost of a trip, Julie paid half of what Mary paid, and Peter paid the remaining $75,
Answered by Penny Nom.
A fraction with a negative exponent in the denominator 2015-04-30
From ellen:
7/ab to the -4 power
Answered by Penny Nom.
A word problem with fractions 2015-04-23
From Cecilia:
if one third of a pole is red, one quarter is white and 15m is black what is the height of the entire pole
Answered by Penny Nom.
A word problem with fractions 2015-04-09
From Lorraine:
If the numerator of a certain fraction is doubled and the denominator is increased by 1, the fraction becomes 1/2.

If the numerator of the original faction is squared and the denominator is decreased by 2, the fraction becomes equal to 1.

Let x be the numerator and let y be the denominator of the original fraction. Write down two simultaneous equation in x and y.

Solve these equations to find two possible values for the given fraction.

Answered by Penny Nom.
Ordering fractions 2015-03-27
From latoya:
Put the following amounts in order from greatest to least 60%,2/3,0.599 explain answer
Answered by Penny Nom.
Olivia and Ray walk to school 2015-03-12
From Rebeccah:
Olivia and Ray walk to school. Olivia walks 1/4 of a mile to school. Her walk is 2/3 of the distance that walks Ray to school. What is the total distance,in miles,that Ray walks to school?
Answered by Penny Nom.
Two equations with fractions 2015-02-26
From Pulane:
Hi math centre I've been trying to solve these equations for days now please help (6/x)-(1/y)=4
(9/x)+1=(-2/y)

Please help me solve them simultaneously
Thank you

Answered by Penny Nom.
Two equations 2015-02-16
From nigel:
2x+1/2y=1
6x-3/2y=21

Answered by Penny Nom.
A word problem with fractions 2015-02-11
From Omokayode:
A vehicle tank was 3/5 full of petrol. When 21 litres of fuel was added it was 5/6 full. How many litres of petrol can the tank hold?
Answered by Penny Nom.
Comparing fractions 2015-02-02
From shealla:
Please help me with my assignment please I'm having trouble answering this one..
Write the larger fraction in each pair..
A. 2/3, 4/7 B. 1/3, 2/5 C. 5/8, 3/5

Answered by Penny Nom.
A trip to Ottawa from Toronto 2014-12-22
From Kristy:
A trip to Ottawa from Toronto will take 4 1/2 hours. Assuming we are 2/3 the way there, how much longer will the trip take?
Answered by Penny Nom.
How do you change a decimal to a ratio? 2014-11-29
From Esmeralda:
How do you change a decimal to a ratio? Express the decimal 0.2 as a ratio.
Answered by Penny Nom.
A 10-foot piece of PVC pipe 2014-10-21
From David:
A plumber has a 10-foot piece of PVC pipe. How many 9/5-foot pieces can be cut from the 10-foot piece ?
Answered by Penny Nom.
Why express fractions in lowest terms? 2014-10-09
From John:
I teach in a Faculty of Education. A colleague in the university asked me about fractions and lowest terms. I am quoting the person below, and would appreciate your insights into the question/thought.

"One I'm struggling with is why (for example) 6/18 is not considered as good an answer to a fraction question as 1/3. The traditional response is that 6/18 is not in lowest terms so the question has not been finished until the fraction is reduced, but what actually makes the lowest terms answer the better one? Is it convention? Is there a way to explain why simplest form answers in fractions are right and and anything else is considered incorrect without alluding to some need for this 'good habit' elsewhere in math or science? Is there a real-life reason?" Thank you for any insights and if you have nothing to offer to this query, that is fine too. I appreciate your consideration. John

Answered by Robert Dawson.
A word problem with fractions 2014-08-01
From nathanielcabalan:
The denominator of a fraction is one more than the numerator. If the numerator is increased by three, the resulting fraction is one more than the original fractional. Find the original fraction.
Answered by Penny Nom.
4/9 of a set of story books cost Rs 630. 2014-07-09
From YASIN:
If 4/9 of a set of story books cost Rs 630. What will the rest of the set cost?
Answered by Penny Nom.
How do I make fractions into decimals? 2014-05-31
From jay:
Hi,
I wanted to know how do I make fractions into decimals example 1 3/8?

Answered by Penny Nom.
A word problem with fractions 2014-05-30
From shadab:
A container of milk is 4/5 full. When 10l milk is poured into it, the container becomes 9/10 full. What is the capacity of the container ?
Answered by Penny Nom.
A word problem with fractions 2014-05-23
From Tanzeela:
There are 1200 pupils in a school. 2/3 of them are girls. 1/4 of the boys are overweight. How many boys are overweight?
Answered by Penny Nom.
How many boards did the carpenter start with? 2014-05-08
From Yanly:
A carpenter has several boards of equal length. he cuts 3/5 of each board. After cutting the boards, the carpenter notices that he has enough pieces left over to make up the same length as 4 of the original boards. How many boards did the carpenter start with?
Answered by Penny Nom.
Fractions and units 2014-04-28
From Aaron:
I can't do this math problem. please help. Find 200m over 1 min times 60 min over 1 hr times 1 km over 1000 m. the choices are a. 300 km/hr b. 12 km/hr c. 300 m/hr and d. 12 m/hr
Answered by Penny Nom.
Simultaneous equations with fractions 2014-04-19
From Maryam:
I looked at your example of simultaneous equations with fractions and applied it to my question from an educate exam papers but I couldn't get it to work. The question is:

x/8 - y = -5/2
3x + y/3 = 13

Answered by Penny Nom.
Ratios 2014-04-18
From Kenneth:
Should ratios contain only whole numbers and not any fractional numbers?

For example, 8:1, 2:5 have whole numbers. The ratios 2.5:10 and 1:4.5 do not have just whole numbers entirely.

If it is incorrect to express a ratio with fractional numbers, why are complex fractions correct?

I thank you for your reply.

Answered by Harley Weston.
Ratios 2014-04-18
From Kenneth:
Should ratios contain only whole numbers and not any fractional numbers?

For example, 8:1, 2:5 have whole numbers. The ratios 2.5:10 and 1:4.5 do not have just whole numbers entirely.

If it is incorrect to express a ratio with fractional numbers, why are complex fractions correct?

I thank you for your reply.

Answered by Harley Weston.
Decimals,fractions and percentages 2014-04-11
From Frances:
Ask an expert to find out their real life usage of your topic... that is the question. meaning what is your real life usage of using decimals,fractions and percentages. This is all part of my math assignment to ask an expert real life usage... so please reply back as soon as possible
Answered by Penny Nom.
3/4lbs of nails fills a container 2/3 full 2014-04-04
From Tyler:
If 3/4lbs of nails fills a container 2/3 full, then how many lbs of nails will fill the container?
Answered by Penny Nom.
A word problem with fractions 2014-03-26
From lois:
a man has a stack of boards, he cuts 3/5 off each board, after he cuts the boards he finds he has enough pieces left to make 4 boards the length of the original boards. how many boards did he start with.
Answered by Penny Nom.
29% as a fraction 2014-03-07
From kylie:
29% into a fraction or mixed number
Answered by Penny Nom.
A word problem with fractions 2014-02-23
From alex:
In a certain fraction the number by n the denominator is greater than the number by3.If 2 is added to both the numerator and denominator ,the fraction is increased by 6/35..Find the fraction.
Answered by Penny Nom.
Simultaneous fractional equations 2014-02-15
From benjamin:
hi math central. benjamin here. during class, i had problem with this topic. normally i wont have problems with math but this topic i just too hard for me. please help i am having exam and test next week on this topic

here is the question:
using substitution method, solve the simultaneous equation.
(x+1)/(y+2)=0.5

(x-2)/(y-1)=1/3

Answered by Penny Nom.
Adding fractions 2014-01-31
From bianco:
2/3+3/4=
9/8-6/8=

Answered by Penny Nom.
A word problem involving fractions 2014-01-23
From Pamela:
John put 1/4 of his flowers in a vase
He gave 1/2 to of the flower to his wife
He gave 1/5 of the flowers to his daughter
He has 8 flowers left
How many flowers did John start with

Answered by Penny Nom.
Fractions and exponents 2014-01-11
From john:
what is the exponent form of 9^4/7^-2 and 6^-3/10^-5
Answered by Penny Nom.
Fractions and square roots 2013-12-04
From arionne:
How do you solve a square root with improper fractions like 121 over 49
Answered by Penny Nom.
Problem solving with fractions 2013-12-03
From Jennie:
Paul has 2/3 as many postcards as Shawn. Shawn has 3/5 as many postcards as Tim. If the 3 boys have 280 postcards, how many more postcards does Tim have than Paul?
Answered by Penny Nom.
0.35 as a fraction 2013-11-28
From Laura:
How do you write 0.35 as a fraction or mixed number?
Laura

Answered by Penny Nom.
Three roommates share the rent 2013-11-23
From Cortlin:
Because of different bedroom sizes, three roommates decide to split the rent this way: Roommate A pays 75% as much as B does, and C pays 40% as much as B does. What fractional part and percent of the rent does each roommate pay?
Answered by Penny Nom.
Adding mixed numbers 2013-11-20
From Kathy:
1 3/4 + 1 2/3= ?

5 1/2 - 2 5/6= ?

Answered by Penny Nom.
Is-5/-11 a positive rational number? 2013-10-30
From tazneem:
Is-5/-11 a positive rational number
Answered by Penny Nom.
An equation with fractions 2013-10-22
From Bill:
1/5n= -9
Solve this equation in simple steps!

Answered by Penny Nom.
Problem solving 2013-10-02
From naomi:
My son having trouble a math problem please help with this question a bag of cough drops contains 36 drops. the fraction of each flavor is shown in the table strawberry 5/12 honey 1/12 cherry 1/6 mint1/9 and lemon 1/4 which flavor is the greast in the bag
Answered by Penny Nom.
A garden path 2013-09-06
From mary:
garden path has 8 white slabs, four of the slabs are one third grey, what area of the path is grey
Answered by Penny Nom.
Ann has a bag of marbles. 2013-07-25
From Carmen:
Ann has a bag of marbles. she gave one half to her friend Tina and then one third to her friend Tom. Now left with 15. How many did she had originally?
Answered by Robert Dawson and Penny Nom.
18/y=30/25 what number does y represent 2013-07-20
From celeste:
18/y=30/25 what number does y represent
Answered by Penny Nom.
150 litres has been drawn from a tank 2013-07-14
From henry:
When 150 litres has been drawn from a tank, it is 3/8 full, how many litres will the tank hold?
Answered by Penny Nom.
{(1+x)^1/3-1/3X(1+x)^-2/3}/(1+x)^2/3 2013-06-17
From STEPHEN:
{(1+x)^1/3-1/3X(1+x)^-2/3}/(1+x)^2/3
Answered by Penny Nom.
Inverting fractions and percentage 2013-05-18
From Carlos:
If I calculate the percentage change between two fractions, say, 1/100 and 2/100, I get 100% change. Now, if I flip denominator and numerator, 100/1 and 100/2, and repeat the calculation, I get a 50% change. Why are the percentages different?
Thank you very much

Answered by Penny Nom.
The percentage of fat in a container of yogurt 2013-05-17
From Katy:
There is a question that I' having trouble with. It says: "A container of yogurt contains 228 g of water, 54 g of carbohydrates, 12 g of protein and 6 g of fat, what percentage of the yogurt is fat?" How do you convert 6 g into percentage?
Answered by Penny Nom.
Cutting three sections of pipe 2013-05-16
From Michael:
A plumber cuts three sections of pipe from a 12’ length of ABS pipe, the lengths of the sections are 33 3/8”, 56 5/8” and 39 7/8”. What is left over from the full length, if the saw cut is 1/8” wide?
Answered by Penny Nom.
Sharing in a car rental 2013-04-09
From Marilyn:
8 persons are planning to share equally the cost of a rental car. If 1 person withdraws from the arrangement and the others share equally the entire cost of the car, then the share of each of the remaining persons is increased by--------------?
Answered by Penny Nom.
Simultaneous equations with fractions 2013-03-31
From Terence:
5/x-6/y=1 17/x+30/y=16 I been spending whole day to solve this question. Would be very grateful if you can help I try The denominator value is a equations term which make is simultaneous equations so hard.
Answered by Penny Nom.
Fractions over fractions 2013-02-22
From Kenssa:
Question from Kenssa, a parent:

Simplify: 3-(7/x)-(6/x^2) / (3/x^2)+(5/x)-2

Answered by Harley Weston.
7x-1/4-1/3[2x -1-x/2]=19/3 2013-02-03
From M:
7x-1/4-1/3[2x -1-x/2]=19/3
Answered by Penny Nom.
Percent as a fractional relationship 2013-01-14
From Kenneth:
What does the following indicate?

"A percent is another way of expressing the ratio or fractional relationship of two numbers."

I know what a ratio is, but the term "fractional relationship" confuses me.

I thank you for your reply.

Answered by Penny Nom.
2 1/4 x 1/8 x 1 3/4 x 12 4/9 x 3 2012-12-21
From Tony:
Ok my question

2 1/4 x 1/8 x 1 3/4 x 12 4/9 x 3

I just can't figure out the question ?

Answered by Penny Nom.
Joe and Chris each bought a six pack of cola 2012-11-01
From Loulou:
Joe and Chris each bought a six pack of cola. Joe gave 2/3 of his away and Chris gave half as many as Joe. how many more colas did Chris have than Joe
Answered by Penny Nom.
Converting a fraction to a decimal 2012-10-20
From A Teacher:
My students want an easy way to convert a fraction to a decimal, can I get one from you?
Answered by Penny Nom.
A word problem involving a fraction 2012-10-12
From Derrick:
If the numerator and denominator of a fraction are both decreased by 1 the fraction becomes 2/3. If the numerator and denominator are both increased by 1 the fraction will be 3/4. Find the original fraction. How to do?
Answered by Penny Nom.
Adding two fractions 2012-09-19
From jeanette:

Question from jeanette, a parent:

can you help me figure this according to BEDMAS I am lost
steps please:
\[\frac{12.75^2}{50} + \frac{13.49^2}{60}\]
I hope this is not too confusing for you it is to me...


Answered by Harley Weston.
What percent of her monthly take home pay are her monthly expenses? 2012-09-19
From tina:
a girls monthly take home pay is 1932.00. her monthly expenses total 1910.00. What percent of her monthly take home pay are her monthly expenses?
Answered by Penny Nom.
Equivalent fractions 2012-09-13
From lily:
if i had 1/3 and i had to find the problem an they gave me 20 what will be the denominator
Answered by Penny Nom.
Adding more than 2 fractions 2012-09-10
From Anonymous:
How do you add or subtract more than 2 integer fractions?

OK, here is an example.

-5/6 + 7/4 + 13/8

Please tell me the steps.

Thanks- grade 9 student

Answered by Robert Dawson and Penny Nom.
Ordering fractions 2012-08-22
From Imani:
I am a 6 th grader and on my homework it asks to order the fractions from least to greatest 11/12, 9/10, 10,/11, 15/16. Please help with solving this question.
Answered by Penny Nom.
Multiples 2012-05-28
From Kenneth:
If I understand correctly , a multiple is a product of two numbers. For example some of the multiples of 6 are 6, 12, 18, 24, 30, etc. I just multiplied 6 by 1, 2, 3, 4, 5, etc.

Are the multiples of a fraction, for example, 2/3, determined in the same way? Are they 2/3, 4/3, 6/3, 8/3, 10/3, etc., or are they instead, 2/3, 4/6, 6/9, 8/12, 10/15, etc.?

Or do fractions have no multiples?

Answered by Penny Nom.
Sharing the profit 2012-05-14
From Mafiza:
A and B started a business by investing $6000 and $8000 respectively. At the end of the year, a profit of $2100 is made. How much amount will A get as his share of profit?
Answered by Penny Nom.
A 12 foot long board 2012-04-05
From Lesley:
If I have a board 12 feet long and am building shelves 2 1/4 feet in length, how many inches are left?
Answered by Jaymi Peterson and Penny Nom.
Fractions and proportions 2012-02-16
From Kenneth:
Hello:

I read the following in an old textbook:

Display the numbers 27, 18, 26, and 39 so as to form a proportion.

Which set of equal fractions is correct?

18/27 = 26/39

18/26 = 27/39

I thank you for your reply.

Answered by Penny Nom.
An equation with fractions 2012-01-06
From Balkees:
3/4(4-8x)=2x-2/3(6-12x)
Answered by Penny Nom.
Adding fractions 2012-01-05
From laurie:
if shaded parts of a circle are 4/7 and the other is 1/3 what fraction is left
Answered by Penny Nom.
Ordering Fractions 2011-11-16
From Chloe:
hey okay im having trouble with a math question. how do you put these fractions in number order
8/14 11/28 3/7

Answered by Jaymi Schmidt.
Ordering fractions 2011-11-16
From Laya:
Order from least to greatest
2/3, 1/2, 5/12

Answered by Jaymi Schmidt.
1 third plus 5 sixths 2011-11-10
From Matt:
1 third plus 5 sixths =
Answered by Penny Nom.
Three men sharing a task 2011-09-27
From Olaniyan:
Three men do a piece of work, the first does 7/15, the second does 5/6 of the remainder while the third does the rest.if the rest done by the third person equals 24 units, find the whole piece of work?
Answered by Penny Nom.
A tank is 3/8 full 2011-09-24
From Olaniyan:
When 150 liters has been drawn from a tank, it is 3/8 full; how many liters will the tank hold?
Answered by Penny Nom.
- 16 x 6 / -2 2011-09-09
From Nadiyah:
i dont understand how to answer this question ;
- 16 x 6 / -2
i still dont understand it with bedmas
/ = divison
please help!

Answered by Penny Nom.
Ordering fractions 2011-08-17
From Myon:
My problem is putting fractions from least to greatest and they are 1/4 2/5 1/3 4/6 1/2 8/9 3/4 1/1 2/6 1/10 thanks
Answered by Penny Nom.
A ratio as a fraction in lowest terms 2011-08-12
From jonecia:
write this ratio as a fraction in lowest terms 707days of 112days?
Answered by Melanie Tyrer.
A linear equation with fractions 2011-07-13
From Simon:
I would like to ask if you would be able to explain : linear equation: 2 1/2 (X-1) - x+3 /3 = 4 , the first step shows multiplying by 2 then multiplying by 3. etc. What I don't understand is where the 2 in multiplying by 2 comes from? ( the three is pretty obvious being x+3 /3 (to get rid of the divided by) ) .
Thanks for your help.

Answered by Penny Nom.
17/20 of way around around an octagon 2011-06-05
From Priscilla:
What side of an octagon is a snail on if it is 17/20 way around? I think it is 6.8? Does that mean it is on side 6 or 7?
Answered by Penny Nom.
pi/2 + 12/13 2011-05-24
From Jen:
If you have (pi/2) and you want to add it to (12/13), how would you go about it?
Answered by Chris Fisher.
Problem solving with fractions 2011-05-21
From emilyd:
I had a packet of sweets. I put 1/5 of the sweets into Container A, 1/3 of them in Container G, 10 sweets in container C and the rest in container D. If the number of sweets in container D is equal to 75% of the total number of sweets in container A and B, how many sweets are there altogether?
Answered by Penny Nom.
(27^r^-1)^-2/3 2011-04-24
From Yung-Ju:
(27^r^-1)^-2/3
Answered by Penny Nom.
x/6 + 4/5 = x/9 2011-04-10
From Khadija:
x/6 + 4/5 = x/9
Answered by Penny Nom.
Write 3 1/5 in decimal form 2011-03-24
From Stephenie:
converting fractions????

3 1/5 into decimal???

Answered by Penny Nom.
Ordering fractions 2011-03-20
From Yasmine:
what's the answer, greatest to least

3/8 1/4 2/3

Answered by Penny Nom.
Ordering fractions 2011-03-01
From amanda:
I am utterly confused my son needs to order 3 different fractions from least to greatest such as 3/6, 5/6 & 4/6 I read your explanation on how to do it but I was still confused is there a more simpler method?
Answered by Robert Dawson.
Factoring with fractions 2011-02-15
From Megan:
Hi there, I'm working on factoring polynomials but this question has me quite puzzled.. im a college student in my first year.

(x^2-a^2/xy)(xy/x+a)

Answered by Penny Nom.
1 and four eights inches 2011-02-01
From Miranda:
WRITING TO EXPLAIN:If a line is measured as 1 and four eighth inches long,explain how you could simplify the measurement?
Answered by Penny Nom.
A mixed number 2011-01-30
From Joyce:
express the fraction as a mixed or whole number 94/5
Answered by Penny Nom.
1/a^2 + 1/b^2 2011-01-19
From robert:
If (a + b)^2 = 81 and ab = 18, find the value of 1/a^2 + 1/b^2 ?
Answered by Penny Nom.
Ordering fractions 2010-12-14
From serita:
1/2,2/4,4/8 how do you do least to greatest
Answered by Penny Nom.
Simultaneous equations 2010-12-05
From ryan:

Question from ryan, a student:

3         4
--   -    --      =     1      (1)
x         y

7       2             11
--   -  --        =   --       (2)
x       y              12


Answered by Chris Fisher and Stephen La Rocque.
Ordering mixed numbers 2010-11-23
From kate:
what are the mixed numbers 5 7/9, 5 1/2, 5 11/18. put least to greatest?
Answered by Penny Nom.
Ordering fractions 2010-11-14
From Alexia:
I am having real trouble with how to order 18/11 , 2/5 ,6/15 from least to greatest can you give me a step by step real easy way to do it?
Answered by Penny Nom.
Simplifying fractions 2010-10-13
From Alice:
14times (9divied3) + 12
______________________
18 divied (26-24)

don't get this.......

Answered by Penny Nom.
4√3 + 2√1/3 2010-09-25
From Michelle:
How do you solve 4√3 + 2√1/3? Please show step by step.
Answered by Penny Nom.
order fractions from least to greatest 2010-09-20
From Kameron:
How do I work this out to find the answer from least to greatest ? 48/8, 12/16, 3/5
Answered by Melanie Tryer.
La racine carrée et l'exposant une demie 2010-09-14
From Alain:
Bonjour. Je cherche une explication sur l'équivalence entre les exposants fractionnaires et les racines nième. Par exemple, comment prouve-t'on que la racine carrée correspond à  l'exposant une demie? merci
Answered by Pierre-Louis Gagnon et Claude Tardif.
An algebraic equation with fractions 2010-09-10
From leah:
If 6/x - (x-1)/2 = 4 then the LCD is 2x, right? so you get 12/2x - (x2-x)/2x = 8x/2x, right? then what do you do with the 2x when you turn it into a quadratic equation?
Answered by Penny Nom.
Mrs. Johns graded 121 math tests. 2010-09-10
From Tums:
Mrs. Johns graded 121 math tests. It took her 3 hours to grade all of the tests. Each test took the same amount of time to grade. About how many test did Mrs. Johns grade in 2 hours?
Answered by Penny Nom.
Can a fraction be an even or odd number? 2010-08-25
From Kenneth:
Can a fraction be an even or odd number? If not , why?
Answered by Stephen La Rocque and Harley Weston.
Ordering fractions 2010-08-24
From COCO:
Hello. I am having lots of trouble with these types of problems. Here is one of them:1/2 1/18 3/6. And I need to order them from greatest to least. Please also explain how to do it.
Answered by Robert Dawson.
An algebraic fraction 2010-06-28
From luis:
3x-2//4-3x=12
Answered by Stephen La Rocque.
Distribution with Fractional Terms 2010-06-11
From Vincent:
Compounded semiannually. P dollars is invested at annual interest rate r for 1 year. If the interest iscompounded semiannually, then the polynomial P(1 + r/2)^2 represents the value of the interest after 1 year. Rewrite this expression without parentheses. Evaluate the polynomial if P=$200 and r = 10%.
Answered by Janice Cotcher.
Who is more extravagant? 2010-06-01
From likhitha:
kunal and ramit have Rs.40 and Rs.50 respectively. Kunal spends Rs.18 and ramit spends Rs.21.Who is more extravagant ?
Answered by Penny Nom.
Algebraic fractions 2010-04-22
From rory:
3x/(x²-64)+4/(x²-6x-16)=
Answered by Robert Dawson and Harley Weston.
If a third is a fourth of a number, what is the number? 2010-04-08
From larry:
If a third is a fourth of a number, what is the number?
Answered by Penny Nom.
Tiling a floor 2010-03-31
From shane:
a floor in a house is 12'6" in width and 10'4" in length. Tiling the floor with each tile 5" on each side. First express the square footage into an improper fraction. Second express the area of each tile in square feet. Third how many tiles needed to tile the floor. Fourth explain how answers relate to real world
Answered by Harley Weston.
What is 8.6597 divided by two thirds? 2010-03-22
From patsy:
what is 8.6597 divided by two thirds
Answered by Penny Nom.
Decimals and fractions 2010-03-16
From fay:
why do we use decimals in place of fractions in everyday life?
Answered by Robert Dawson.
Ordering fractions 2010-02-26
From jessie:
I'm having alot of trouble with orderig fractions from least to greatest! I'm in 5th grade....and my problem is, 5/8,1/2, and 3/4
I need to know HOW to order them.........can anyone give me a simple explanation?

Answered by Penny Nom.
Ordering fractions 2010-02-26
From Brianna:
I'm a seventh grader and we're learning about fractions. How would you list the numbers in order from least to greatest?
The options:
A. 9/14 , 17/28 , 11/21
B. 17/28 , 9/14 , 11/21
C. 9/14 , 11/21 , 17/28
D. 11/21 , 17/28 , 9/14

Answered by Penny Nom.
Is 0.1234567891011.... periodic? 2010-02-23
From Adriana:
0.1234567891011.......Is this fraction periodic ?
Answered by Penny Nom.
15 men can do a piece of work in 7 days 2010-02-20
From Kenneth:
If 63 books cost $126, what will 125 books cost?

If 15 men can do a piece of work in 7 days, in how many days can 21 men do the same work?

Answered by Penny Nom.
An algebraic equation with fractions 2010-02-19
From Ingrid:
What is the solution set of the equation x over x plus 4 = 1 over x plus 3 = 28 over x to the 2nd power - x- 12?
Answered by Penny Nom.
1/6 x 5 x 12 = what 2010-02-19
From dana:
1/6 x 5 x 12 = what
Answered by Lorraine Dame.
Least common denominator 2010-02-13
From Priscila:
3/8 + 4/5 + 7/3 + 9/10 = ?

Thank you for your assistance.

Priscila

Answered by Penny Nom.
Combining fractions 2010-02-10
From Nick:
Combine the fractions

2m/t + 5/mt

Answered by Penny Nom.
Mixed numbers and multiplication 2010-02-08
From Nick:
Can you explain how this works, I have been going at it for hours and have a test friday and just cant get my head around the question
3 1\4 * 1/7 * 8

I have to express in lowest terms

Answered by Penny Nom.
Mixed numbers 2010-01-13
From Linh:
Perform the indicated operations:
6 1/4 + (- 2/3) x 1 1/2

Answered by Penny Nom.
Improper fractions 2010-01-12
From Blanca:
Between which two consecutive natural numbers does the improper fraction 57/56 lie?
Answered by Robert Dawson.
A linear equation with fractions 2009-12-31
From Michelle:
How would I solve: (-2)(-1/2x + 5) = (-2)(8)?
Answered by Penny Nom.
Decimal equivalent 2009-12-13
From Asia:
In a forest,it rained 36 out of 90 days. What is the decimal equivalent of 30 out of 90 days
Answered by Penny Nom.
A number base that's not a positive integer 2009-12-11
From Nick:
Is it possible for a number to have a base that's not a positive integer? Base 1 gives a result that's meaningless but possible. I have no concept of a base 0, a negative base, or a fractional base, never mind any other base. I think there isn't any base that's not a positive integer, but, knowing that math keeps jumping ahead and sometimes has inventions before anyone knows how to exploit them, I think I'd better ask.

Thank you.

Nick

Answered by Robert Dawson and Claude Tardif.
A rate of 67 per 1000 2009-11-10
From ANTHNONY:
if the rate is 67 per 1000 the fraction is 67/1000 how do I present the expanded form
Answered by Penny Nom.
Sarim has $1 in coins. 2009-10-27
From Elizabeth:
Here is the question asked: Sarim has $1 in coins. One-fifth of the coins are dimes, two-fifteenths are nickels, and two-thirds are pennies. Tell how many of each coin he has. I am not sure how to start this problem.
Answered by Penny Nom.
What fraction of my goal has been raised? 2009-10-13
From sally:
If my goal is to raisel $300 and I have only raised $240 what fraction of my goal has been raised
Answered by Penny Nom.
How many gallons does it contain when it is 1/6 full? 2009-10-13
From Bren-Nesha:
A tank contains 200 gallons when it is 4/5 full. How many gallons does it contain when it is 1/6 full? I already know the answer but i would like to know how to arrive at the answer.
Answered by Penny Nom.
Comparing fractions 2009-09-28
From TeeTee:
I am in 6th grade. I have to order these fractions from least to greatest: 3/4, 2/5, 5/8, 1/2.....How do I order them if I dont know where to start, and how would I find these answers...Please Help! :)
Answered by Penny Nom.
Repeating decimal to fraction 2009-09-18
From joan:
how can i answer it easily to convert terminating decimal to fraction? example of this is that, convert ...143143143... to fractional form
Answered by Robert Dawson.
Ordering fractions 2009-09-15
From stacy:
I need help with listing fractions from the least to greatest. last problem of math is 5/6,7/9,23/27,13/18?
Answered by Penny Nom.
Divide and reduce to lowest terms. 2009-09-06
From Karen:
Divide and reduce to lowest terms. Use the cancellation technique as needed 4 2/3 divided by 12 14/3 divided by 12/1 not sure wheer to go from here with the cancellation technique
Answered by Penny Nom.
Comparing Fractions 2009-08-20
From smilegrlal:
please order these number in least to greatest 7/8, 5/9, 2/3
Answered by Leeanne Boehm.
Portion of Work 2009-07-17
From Ellen:
A girl can shampoo the dog, clean his ears, and clip his nails in 45 minutes. Her sister can do it in 60 minutes. How long will the job take it they work together?
Answered by Stephen La Rocque.
Fractional Exponents 2009-07-07
From bob:
2^5/2 - 2^3/2
Answered by Janice Cotcher.
Solving an Algebraeic Equation with Fractions 2009-06-04
From olivia:
solve for x (3x-1)/4 + (x+3)/6 = 3
Answered by Janice Cotcher.
The quotient of 5/8 divided by7/4 2009-05-25
From tammy:
find the quotient of 5/8 divided by7/4
Answered by Stephen La Rocque.
Equivalent fractions 2009-05-22
From Jocelyn:
give two other ratios that are equal to each

example: 6/9

Answered by Penny Nom.
(x + 1) / 3 + (x + 2) / 7 = 2 2009-05-22
From MzDonna:
Question from MzDonna, a parent:

We can't seem to get an answer for this problem:
X + 1      X + 2
_____ + _____   = 2

   3              7

The book gives the answer as 10 ; But, we struck. Please help us.
                                               ___
                                                21       

Thanks in advance for your assistance.
Answered by Stephen La Rocque.

Fraction Word Problem 2009-05-13
From Sonya:
Debbie wants to eat 1/6 of her 12 pieces of candy. How many pieces did she eat ?
Answered by Janice Cotcher.
How many eighths are there in 1.25? 2009-05-06
From Tamara:
How many eighths are there in 1.25?
Answered by Penny Nom.
3h+10 divided by 2 +9=20 2009-05-05
From CJ:
what is the solution for the equation 3h+10 divided by 2 +9=20
Answered by Penny Nom.
Fractions of fractions 2009-05-02
From Sonya:
9/10 of the students came to school today and 8/9 of the students brought their lunches how many students brought their lunches?
Answered by Stephen La Rocque.
Change 5/6 into a percent 2009-04-15
From Shelly:
My question is when it says change 5/6 into a percent
Answered by Penny Nom.
A fraction in its simplest form 2009-04-02
From Michael:
I'm in 4th grade and need to express decimals as a fraction in its simplest form. Is there a step by step method to figure out?

ex 0.64 = 64/100 = ?

Answered by Robert Dawson.
Fractional part 2009-04-01
From Galina:
What fractional part of eight is eight hundredths
Answered by Stephen La Rocque.
More on the square root of 0.75 2009-03-30
From Blaine:
I read your response to How is the square root of 3/4 is greater than 3/4?

What I'm hoping for is a way for my students to use their own experience and number intuition to be able to make sense of the issue. As soon as my kids see "if y is this and x is this then..." their little eyes glaze over. Unfortunately, I can't come up with a way myself. Thank you for your help.
Answered by Penny Nom and stephen La Rocque.

Dividing by fractions 2009-03-25
From Mitch:
-2 divided by -2/3 = ??
Answered by Penny Nom.
Comparing positive and negative fractions 2009-03-24
From Christyrose:
Hello. I am a sixth grade math student and i need help on how to compare negative and positive fractions.

Here is is:
3/4 -2/3 7/10 -5/6

I'm not sure how to do the negative parts because i was absent for a couple days. Thank you!

Christyrose

Answered by Penny Nom.
0.0007 as a percent 2009-03-18
From Shelly:
I need to know how to divide 0007 by 100 and soon! and I have no idea how I can get 100 to go into a number like that! here's the question this problem originated from:

change the decimal into a percent:
0.0007 _________

How do I change THAT number into a percent?

How do I change 12.455 and 92.348 and 29.005 into percents as well? I know how to change numbers like 0.6 and 0.70 into percents but not the numbers I told you I'm having problems with. Please someone explain this to me and also please focus mainly on the first number I asked about but don't forget the others please. Please reply ASAP I need to know this in about a week or less!

Answered by Penny Nom.
66 2/3% 2009-03-10
From Traci:
Please convert 66 2/3% into a decimal and a fraction
Answered by Penny Nom.
A taxi charges .20 every 1/9th of a mile 2009-03-03
From ronald:
i need to know if a taxi charges .20 every 1/9th of a mile and i have to go 6.55miles how much would that be?
Answered by Robert Dawson.
A fraction with negative exponents 2009-02-14
From Gabriel:
What does the line seperating the top from the bottom mean? what does it tell you to do and what is the answer to the question?

Question 3

Simplify. Write the answer using only positive exponents. Assume all variables represent nonzero numbers.

(x^-5)^-4(x^-1y)^3
________________
(xy^4)^3


Answered by Penny Nom.
What percent is 314.70 of 693.20? 2009-02-11
From steph:
what percent is 314.70 of 693.20
Answered by Penny Nom.
A company fish fry 2009-02-04
From shaelisa:
At a company fish fry, ½ in attendance are employees. Employees’ spouses are 1/3 of the attendance. What is the percentage of the people in attendance who are not employees or employee spouses?
Answered by Robert Dawson.
3/4 0f what=75 2009-01-22
From tyree:
3/4 0f what=75
Answered by Harley Weston.
Problem solving with fractions 2009-01-16
From Linda:
Sam and Mary each owned one-half stock in a printing company. Sam sold 2/5 of his stock to Mary. What fractional part of the printing business does Mary now own?
Answered by Stephen La Rocque.
Ordering fractions 2009-01-14
From Shea:
I am a special education teacher, who is trying to to show my students how to order fractions. I found your response to this questions while looking for information. I need a visual to explain to my students and myself on how and why you multiply the fractions together.
Answered by Harley Weston.
Order from least to greatest 2008-12-16
From staci:
order from least to greatest on a number line,
7/10 3/5 5/10

Answered by Robert Dawson.
Decimals and fractions used in our daily lives 2008-12-04
From josh:
i have a projecct due friday and i need to know, how are decimals and fractions used in our daily lives. i am having trouble coming up with ideas. i need seven more that dont involve money or recipes. please help me.
Answered by Harley Weston.
Fractions of fractions 2008-11-26
From DIANNE:
Question from DIANNE, a student:

5(16-9) divided by 7x2+14 top part of equation
_____________________________________
12-3x2 bottom part
Answered by Penny Nom.

An algebraic exercise with fractions and radicals 2008-11-18
From Rubén:
Question from Rubén, a teacher:

I have the ecuation:

U = [yb / (ab+a^2)]^1/2 + [ya / (ab+b^2)]^1/2

I know for sure this reduces to

U = [y (1/a + 1/b)]^1/2

but I cannot find a way to get into that result!

Thanks

Answered by Rubén Osuna.
How do I compare fractions and decimals? 2008-11-18
From Alexis:
How do I compare fractions and decimals ?
Answered by Penny Nom.
How do you solve " (x-4)^3/2 = -6 "? 2008-11-17
From Jim:
How do you solve " (x-4)^3/2 = -6 " ?
Answered by Penny Nom.
A repeating decimal 2008-11-07
From mike:
what is 0.0028282828 recurring as a fraction?
Answered by Penny Nom.
Algebraic equations with fractions 2008-11-07
From John:
Solve each equation. Check each solution.
1/8+5x/x+2=5/2
10/2y+8 - 7y+8/y(squared)-16 = -8/2y-8

Answered by Harley Weston.
Job applicants 2008-11-03
From dave:
Three-fourths of the applicants for a position had previous experience. If 144 people applied, how many had previous experience? How many did not have previous experience.
Answered by Penny Nom.
A homework problem 2008-10-28
From Shawn:
I'm checking my son's homework and we disagree on the solution. The problem reads: 135mn^4 (n to the 4th power) over 50n^2 (n squared) I think the answer is: 27mn^2 (squared) over 10 Can you help?
Answered by Penny Nom.
Order the #s from least to greatest 2008-10-24
From morgan:
order the #s from least to greatest

18 7/12, 18 2/3, 183/4, 18 1/3

Please help
Morgan

Answered by Stephen La Rocque.
equivalent fractions 2008-10-06
From marcella:
a fraction equivalent to 16/20
Answered by Harley Weston.
Algebraic fractions 2008-10-06
From Kayla:
(x^2-9)/x times (x^3-4x)/(x^2+5x+6)
Answered by Harley Weston.
Simplify [[1/(x+3)]-1/3]/x 2008-09-19
From diana:
How does [[1/(x+3)]-1/3]/x simplify
Answered by Harley Weston.
Adding fractions 2008-09-17
From amanpreet:
please can you help me to simplify:

2(c-3) / 7 + 3(4c-2)/2

Answered by Penny Nom.
Putting fractions in order 2008-09-11
From kaby:
whats the answer for this? put it from least to greatest: 1/4, 1/9, 1/8
Answered by Penny Nom.
Repeating fractions 2008-09-09
From Juli:
My teacher recently put my math class to the test... We were deiscussing repeating fractions and she asked us to find out what the bar over a repeating decimal is called. I found out it was called the vinculum. But she also said to find out what the number under the vinculum was called. I can't seem to find it anywhere.
Answered by Penny Nom.
How would put .12 with 2 repeating into a fraction? 2008-09-09
From Savanna:
How would put .12 with 2 repeating in fraction?

Savanna!

Answered by Penny Nom.
Simplify ((5/7)/(5/14)) + (3/4) 2008-09-06
From Emal:
How do you simplify completely: ((5/7)/(5/14)) + (3/4)?
Answered by Penny Nom.
A mixed number 2008-09-04
From irena:
Need help writing this fraction as whole or mixed number 124/6
Answered by Penny Nom.
A fraction word problem 2008-08-29
From Sylvia:
The denominator of a fraction is 1 less than 4 times the numerator. If the numerator is doubled and the denominator is increased by 6, the value of the resulting fraction is 2/5. Find the original fraction.
Answered by Penny Nom.
Ordering fractions 2008-08-25
From Ghita:
Order these distances from least to greatest
1/2 , 5/8 , 3/4
What is a quotient?

Ghita

Answered by Penny Nom.
Convert Repeating Decimal into a Fraction 2008-08-25
From Craig:
Can you help me to convert: 0.444444 (repeating decimal - base 10) to base 9?
Answered by Janice Cotcher.
The square root of a fraction 2008-08-24
From Lauren:
How do you solve square roots of fractions? Does the format change if it is either a proper or an improper fraction?

Ex. the square root of 1/4 or the square roots of 80/25

Answered by Penny Nom.
1/X-2 + 3/X+3 = 4/X^2+X-6 2008-08-06
From Molly:
Hi, I'm beginning pre-calc this semester, and for practice over the summer, the teacher has given us a review packet with algebra 2 stuff. Well, I've forgotten most of the fraction stuff on it. Would you please help? Here's the question:

1/X-2 + 3/X+3 = 4/X^2+X-6

Answered by Penny Nom.
Egyptian fractions 2008-07-09
From vivek:
An Egyptian fraction has numerator equal to 1, and its denominator is a positive integer. What is the maximum number of different egyptian fractions such that their sum is equal to 1, and their denominator are equal to 10 or less?
Answered by Janice Cotcher.
x=(-14x+16)/x-8 2008-06-11
From Kory:
Solve the equation.
x=(-14x+16)/x-8

Answered by Penny Nom.
Which fractions should go next? 2008-06-11
From Sharon:
Please help me with these fractions
1/4, 2/3, ___, 5/6
which fractions should go next
A.1/2
B.1/3
C.3/4
D.11/12

Answered by Janice Cotcher.
What is 6 over 8 as a percentage? 2008-06-11
From stephen:
what is 6 over 8 as a percentage?
Answered by Penny Nom.
Converting a decimal to a fraction 2008-05-19
From judy:
please explain how to get from the decimal number .3437 to the fraction 11/32
Answered by Leeanne Boehm.
Ratios and fractions 2008-04-20
From Alissa:
What is the formula for writing the ration of two numbers as a fraction?
Answered by Leeanne Boehm.
What is 300% as a fraction? 2008-04-14
From Carli:
what is 300% as a fraction
Answered by Stephen La Rocque.
What fraction of 45 is 10? 2008-04-06
From Scott:
45 x fraction? = 10
Answered by Penny Nom.
Common fraction to decimal fraction 2008-04-01
From chadwick:
how do I turn 3/32 into a decimal
Answered by Penny Nom.
Operation on the mixed numbers 2008-03-26
From bradley:
1) Perform the indicated operation on the mixed numbers below; write answer in simplest form:
8 1/3 – 2 1/4

2) Perform the indicated operations on the mixed numbers below; write answer in simplest form; note;: "•" denotes multiplication:
2 1/2 • 3 2/3 • 5 3

Answered by Stephen La Rocque.
The decimal equivalent of a fraction 2008-03-24
From Mario:
What is the decimal equivalent of the fractions 5/8?
Answered by Penny Nom.
Repeating decimals 2008-03-10
From Blaine:
Is it possible to put a repeating decimal number into a calculator to solve a problem?

EX: Write 39.3939... as a fraction.

Answered by Penny Nom.
Comparing fractions 2008-02-21
From Melissa:
Hello i am a sixth grader, On my homework sheet there is the question : 5/45 _ 9/81 But it says to Compare the fractions. Use <,>, or = and i find it difficult to answer, please help me find a simple way to answer questions like this.
Answered by Stephen La Rocque and Penny Nom.
Algebraic fractions 2008-02-21
From sergio:
how to simplify
(x^2 +5x+6)/(x^2 - 4) x (x^2 -5x+6)/(x^2-9)

Answered by Stephen La Rocque.
Subtracting two algebraic fractions 2008-02-17
From Willie:
Subtract and express the answer in simplest form. 5x-3/6-x+3/6
Answered by Penny Nom.
Sum and difference of cubes 2008-01-30
From Amanda:
It has been a really long time since I was in Algebra and I can't remember how to factor cubes such as x^3 +81 or subtracting/adding fractions with variables such as [1/(x+h)+2]-[1/x+2]. Please help!!!
Answered by Penny Nom.
Converting decimal fractions into common fractions 2008-01-29
From Donna:
Question from DONNA:

I need to change 6.9375 by 10.0625 into inches.

Answered by Harley Weston.
18/95 x t =3 3/5 2008-01-28
From emily:
18/95 x t =3 3/5
Answered by Penny Nom.
A fraction comprised as 2 numbers having the same units 2008-01-27
From nancy:
term=a fraction comprised as 2 numbers having the same units
Answered by Stephen La Rocque and Harley Weston.
A developer subdivides a plot 2008-01-25
From Katherine:
A developer buys a plot of land that is 48,000 sq.feet. She sets aside 1/2 of it to bulid a house for herself, then divides the remaining half into 3 equal plots. Find the area of each lot.
Answered by Penny Nom.
Change a mixed fraction into a decimal 2008-01-23
From Qiana:
I need to know how to change a mixed fraction into a decimal. the mixed fraction is 9 1/2
Answered by Penny Nom.
A word problem 2008-01-18
From Marouf:
How can you make a word problem for 6 2/3 and solve the word problem? I'm having trouble. Please help
Answered by Stephen La Rocque.
A club made a quilt 2008-01-17
From joanna:
math question is club made a quilt 16 1/2 square feet 2/3 of the quilt was red how many square feet were red ?
Answered by Penny Nom.
1.2 as a fraction 2008-01-12
From Confused:
i have confused my brain and its driving me crazy i am trying to write 1.2 as a fraction but each time i do it i come out with a different number please help
Answered by Penny Nom.
Partial fractions 2008-01-06
From taiwo:
i copied the question below from john bird's higher engineering mathematics fourth edition (pg 19) after solving all the partial fraction questions from the book except this:

x^3 + 4^2 + 20x - 7 over (x-1)^2 (x^+8). i will appreciate it if you help me out so i can know whether their answer is correct. thanks alot

Answered by Harley Weston.
Ordering fractions 2007-12-31
From MARLYN:
Which list shows number in order least to greatest?

A. 07;2/3. 1/5;0.6

B 1/5;06; 07;2/3

C 1/5; 06; 2/3;0.7

D 2/5;0.6; 1/5;0.7

Answered by Penny Nom.
Adding fractions 2007-12-31
From Lisa:
As an educator in Adult Education, I am preparing a young man for a US Government test. The equation I have trouble with is explaining the following equation. It asks: If it takes A 3 days to dig a certain ditch, whereas b can dig it in 6 days, and C in 12, how long would it take all three to do the job? I know the equation is 1/3 + 1/6 + 1/12 and it equals = 7/12 but I don't understand how it equals 7/12. This simple math eludes me. So the question is not the final answer of 1 and 5/7 of days but where one comes up with the 7/12?
Answered by Penny Nom and Claude Tardif.
Improper fractions and mixed numbers 2007-12-12
From Selah:
How do I change an improper fraction to a mixed number?
Answered by Penny Nom.
Sharing a cake 2007-12-03
From Nikheel:
Amar baked a cake. john ate 1/6 of the cake. susan ate 1/5 of what was left. chan ate 1/4 of what was left. cindy ate 1/3 of what was left. luigi ate 1/2 of what was left. how much of the original cake was left.
Answered by Stephen La Rocque.
A percent as a fraction 2007-11-24
From Abby:
write 43.75% as a fraction or mixed number in lowest terms.
Answered by Penny Nom.
Ordering fractions 2007-11-23
From Joy:
what are the orders of these fractions?
2/3, 1/2, 1/4, 2/10, 3/4, 1/3, 9/10, 2/2.

Answered by Penny Nom.
Red balls and blue balls 2007-11-18
From Md:
In a group of 15, 7 have Red Balls, 8 have Blue Balls and 3 have neither. What fraction of the group has both Red Balls and Blue Balls?
Answered by Stephen La Rocque.
A variable over a number 2007-11-15
From Silver:
3/8x - y/2= 1

I do not understand what you do with the variable over a number.

Answered by Penny Nom.
Ordering fractions 2007-11-15
From DEL:
Hi. I feel really stupid ! I'm a mature student and i have completely forgotten How to find out the order of fractions from largest to smallest. I Have been put this poser; 7/4...1/6....7/2 Can you please tell me what is the largest and lowest? Will be very grateful....thank you
Answered by Gabe Potter.
How do you convert eleven fifteenths into a decimal? 2007-11-14
From Lauren:
How do you convert eleven fifteenths into a decimal?
Answered by Penny Nom and Stephen La Rocque.
Whole numbers and improper fractions 2007-11-13
From Jennifer:
I would like to know how you convert whole numbers into improper fractions
Answered by Melanie Tyrer.
Equivalent Fractions 2007-11-07
From Marlene:
find equivalent fractions with a denominator of 8 1/2
Answered by Stephen La Rocque.
Ordering fractions 2007-11-06
From Mene:
this is the weight of three packages 4/5 kilograms 7/10 kilograms 1/2 kilograms order these weights from least to greatest
Answered by Penny Nom.
A decimal to a fraction 2007-11-01
From Lisa:
i am trying to turn a decimal into a fraction. my example is to turn 10.5 into a fraction at its lowest form
Answered by Penny Nom.
Write each frequency as a decimal 2007-10-31
From Jazzy:
Twins are born once in every 89 births. Identical twins are born 4 times in every 1000. Triples are born once in every 6900 births. write esch frequency as a decimal and order the decimals from least to greatest frequency.
Answered by Penny Nom.
I need to order 3/11, 1/8, 2/9 from least to greatest 2007-10-19
From Andrew:
I need to order 3/11, 1/8, 2/9 in least to greatest.
Answered by Penny Nom.
How far away is the town? 2007-10-17
From georgia:
You are driving to town 15 miles away. If you have already driven 3 1/4 miles, how far away is the town.
Answered by Penny Nom.
Equivalent fractions 2007-10-03
From kiki:
I want to know how to this equivalent decimal for 6.0250.
Answered by Stephen la Rocque and Penny Nom.
Equivalent mixed numbers 2007-10-02
From negra:
an average-sized person can burn about 6 1/2 calories a minute while ridinng a bike. Which of the following is equivalent to that amount?
a) 1 2/2

b) 5 6/2

c) 6 2/4

d)6 2/6

Answered by Stephen La Rocque and Penny Nom.
y squared over 3 times 8 over y 2007-09-30
From John:
y squared over 3 times 8 over y
Answered by Stephen La Rocque.
Algebraic fractions 2007-09-21
From Yvonne:
Would you please help me on the following 2 questions?

1. Given that 3x-5y / 2x-y = 2/3, find the value of 2x/5y.

2. If 3x-5y / 7x-4y = 3/4, find the value of x/y.

Answered by Penny Nom.
Solving an equation with fractions 2007-09-20
From Len:
I am having a brain cramp or maybe I just forgot some basic math, but I am having trouble solving for "r" in your truncated cone example where r/(r+w)=r/R or r/(r+282)=911/1728. Could you refresh my memory by showing the steps to solve for "r"?
Answered by Harley Weston.
A fraction that cannot be simplified 2007-09-17
From Kevin:
Make a fraction that cannot be simplified and has 24 as its denominator?
Answered by Stephen La Rocque and Chris Langdon.
An equation with fractions 2007-09-16
From Stuart:
4/5+3x/8=x/4-1/10
Answered by Victoria West.
1-4/square root 3 2007-09-12
From Prudence:
How to solve this question?
1 - 4/√3

Answered by Penny Nom.
A sequence of fractions 2007-09-10
From mitch:
find an expression for T(n), the nth term of the sequence
1/5, 3/8, 5/13, 7/20, 9/29

Answered by Stephen La Rocque.
24 * 1/6 2007-09-09
From Laurie:
24 * 1/6

I don't no where the 24 goes and if the number must be the same to devide into the numerator and the denominator . Thankyou for your time.

Answered by Penny Nom.
0.24 1/2 2007-09-06
From Kenneth:
The fraction, 1/2, in 0.24 1/2 occupies the 1/100 place along with the 4.
If the fraction, 1/2, is changed to a decimal, as in 0.245, the last 5 in 0.245 occupies the 1/1000 place.
Why doesn't the 1/2 in the decimal 0.24 1/2 occupy the 1/1000 place (0.001) instead of the 1/100 place?

Answered by Stephen La Rocque.
Quotient of proper fractions 2007-09-06
From Tim:
The question asks me to find a counter example for: the quotient of 2 proper fractions is a proper fraction
Answered by Stephen La Rocque.
A sequence of fractions 2007-09-05
From Arjun:
1/2 3/5 5/8 7/11- need to find the nth term.

I did search the data base & found one for fractions but what I want to know is when calculating nth term for the denominator in the example give in your database how do we get (n-1)? When we deduct the actual term with the one that is in the table give in your example it is more that one. Could you please explain solving the above example?

Answered by Penny Nom.
Equivalent fractions 2007-08-31
From Darlene:
What is the equivalent decimal for 0.3 and 6.53
Answered by Penny Nom.
Fractions, ratios and percentages all mixed together 2007-08-29
From Charon:
Example:
x : 1/4% :: 9 3/5 : 1/200

Answered by Stephen La Rocque.
Counting stitches and changing units 2007-08-28
From Lim:
Jack is making stuffed toys. Each toy needs 360 stitches. Ten stitches take 15 seconds.One toy needs 57cm of cotton for stitches. How many centimetres of cotton does Jack use in an hour?
Answered by Stephen La Rocque.
Converting a repeating decimal to a common fraction 2007-08-22
From isabelle:
how do you turn 6.333... into a fraction in simplest form?
Answered by Stephen La Rocque and Penny Nom.
Adding algebraic fractions 2007-08-14
From John:
Ive completely forgot anything to do with the subject mentioned, so my question is straight to the point..

I need to know how to do the following problem (Preferably do not give me an answer though) (k/3k-8) - (4/k+2)

Answered by Penny Nom.
What fraction of the letters of the alphabet is each word? 2007-08-02
From Clayton:
How do I figure this out, the math question has the answers but I have no idea how to get it. my Mom can't figure it out either.

What fraction of the letters of the alphabet are each word
Man = 3/26
Glasses = 7/26
Integrity = 1/4
computer = 2/9

Answered by Paul Betts and Harley Weston.
Simplifying an algebraic fraction expression 2007-07-25
From Jessica:
How do I simplify b/(b2-25) + 5/(b+5) - 6/b?
Answered by Stephen La Rocque.
What fraction of the company does Mary own? 2007-07-18
From Bridget:
Sam and Mary each owned one-half stock in a printing company. Sam sold 2/3 of his stock to Mary. What fractional part of the printing business does Mary now own?
Answered by Stephen La Rocque.
Fractions of fractions 2007-07-18
From Bridget:
Sam and Mary each owned one-half stock in a printing company. Sam sold 2/3 of his stock to Mary. What fractional part of the printing business does Mary now own?
Answered by Penny Nom.
Simplifying complex denominators 2007-06-21
From Krys:
How do I simplify completely? ((4+i ) / (3+i )) - ((2-i ) / (5-i ))
Answered by Stephen La Rocque.
Simplifying a quartic rational expression using long division 2007-06-14
From Megan:
x+2/12x^4+17x^3+0x^2+8x-40=
Answered by Stephen La Rocque and Penny Nom.
Converting dollars to gallons 2007-06-12
From Cynthia:
How do I solve this problem? If a family uses .20 per 100 gallon of water. How many gallons would $10.00 cost, divided amoung 3 people? Thanks, Cynthia
Answered by Stephen La Rocque.
Comparing and ordering fractions 2007-06-12
From virgil:
Order the fractions
2/5, 2/3, 2/7

Answered by Penny Nom.
John ate 1/4 of a cake 2007-06-01
From Jemina:
John ate 1/4 of a cake, Maria ate 1/4 of the remaining cake. What was the unit or whole in John's case? In Maria's case? How much of a full pan was there in each case?
Answered by Penny Nom.
Fractions, decimals and profit 2007-05-29
From fatima:
how can i change a fraction into a decimal and how do i change a decimal into a fraction................
anther question please what does making a profit have to do with percentage change!!!!!!

Answered by Penny Nom.
Ordering fractions 2007-05-08
From Mary:
Can you place these in order? 1/8, 15/64, 15/16, 7/32
Answered by Stephen La Rocque.
Fractions and mixed numbers 2007-05-08
From Amanda:
I am having truble with this problem: WRITE THE FRACTION 8/3 AS A MIXED NUMBER
Answered by Stephen La Rocque.
LCD 2007-04-29
From Jackson:
I am having trouble getting the LCD for the following question. 1 3/8 x 8/9=
Answered by Penny Nom and Melanie Tyrer.
Simplifying a rational expression 2007-04-29
From Tamika:
X^2 + 10X + 24
______________
X + 6
SIMPLIFY EACH RATIONAL EXPRESSION (IF YOU CAN'T SIMPLIFY IT, WRITE ALREADY SIMPLIFIED)

Answered by Stephen La Rocque.
Exponential form 2007-04-16
From Cassia:
I was just wondering, how do you write the squared root of 7x(to the 5th) in exponential form? If you could help explain that I'd be grateful. Thanks
Answered by Brennan Yaremko.
Fractions! Fractions! Fractions! 2007-04-16
From Maria:
Why are fractions important for:
1) The study of Mathematics
2) Real life

Answered by Haley Ess.
Who has the most pecans? 2007-04-10
From Oreste:
Julian gather 3/5 of a bag of pecans Nikki filled 5/7 and Brian 5/9 who filled the most pecans.
Answered by Stephen La Rocque.
Simplify (1/3 + 1/3x) / (1/x + x/3x) 2007-04-04
From Farina:
how do i simplify (1/3 + 1/3x) / (1/x + x/3x)
Answered by Penny Nom.
Understanding fractions and their equivalents 2007-04-02
From Gail:
How can I get fourth grade students to understand fractions and their equivalents?
Answered by Diane Hanson.
2x/3+x-5/4=1/6 2007-03-10
From Nomvuyo:
2x/3+x-5/4=1/6
Answered by Penny Nom.
4th grade equivalent fractions 2007-03-07
From Raymond:
5/8=?/16 ?=?
Answered by Melanie Tyrer and Sara Ulmer.
Adding fractions 2007-02-27
From real:
A person spends 3/7 of her monthly earnings on rent and 1/7 on food. How Much does she spend on food and rent? how much does she have left? How do u work out these questions
Answered by Pam Fowler.
Equations with fractions 2007-02-06
From Juan and Derek:
3/4-x=1/2
3/4+1/5x+-1/3=4/5x

Answered by Penny Nom.
Fractional interest in property (fractions of fractions) 2007-01-19
From Threasa:
Please explain how to solve this problem. Someone inherits 1/2 of 1/3 of 1/4 of 1/8 of 8/8 interest in 100 acres of land would equal what. Please show show in detail how to fiqure this answer. I will be working with a lot of fractional interest.
Answered by Stephen La Rocque.
Comparing two fractions 2007-01-18
From Kayla:
Why does eight over twelve compared to one half work when you use cross multiplication.
Answered by Penny Nom.
Fact families with fractions 2007-01-10
From Joe:
I am trying to help my son with a math problem dealing with fact families and fractions. The question only gives 2 numbers 1/16 + 1/12 and 5/4 - 4/5. Can you help me understand how fact families work with fractions?
Answered by Penny Nom.
How much money is that? 2007-01-05
From Stephanie:
The question is asking fill in the whole box. It is set of 15 nickels. It says circle 2/5 of the nickels? The next question ask How much money is that?
Answered by Penny Nom.
Ordering farctions 2007-01-05
From Samantha:
I am having trouble ordering these fractions: 2/15, 1/5, 3/5

And also would like to know what is $6.67 divided by 8 rounded to the hundredths place

Answered by Penny Nom.
Order the fractions from least to greatest 2007-01-04
From Justin:
I am a sixth grader, and I am having trouble with the last question in my homework assisgnment. 1/6, 2/5, 3/7, 3/5!
Answered by Stephen La Rocque and Penny Nom.
Multiplying fractions 2006-12-09
From Nicole:
When you are Multiplying Fractions why is the answer always smaller?
Answered by Stephen La Rocque.
The fraction 2/3 is found between which pair of fractions on the number line? 2006-11-14
From Anita:
How can i find the answer? What is the formula? The fraction 2/3 is found between which pair of fractions on the number line? A.3/7 and 4/8 B. 1/2 and 3/7 C. 6/7 and 9/8 D. 3/5 and7/9
Answered by Steve La Rocque and Diane Hanson.
How do we use fractions in every day life? 2006-10-19
From Tori:
I am doing fractions in class and we need to write a paper about them. So my question is:"how do we use fractions in every day life?"
Answered by Claude Tardif.
How many acres in the following land description? 2006-10-18
From Tammy:
How many acres in the following land description? south 1/2 of the northwest 1/4 of the southwest 1/4 of a section
Answered by Stephen La Rocque.
What is 1.7142857143 as a fraction? 2006-10-01
From Lizzy:
What is 1.7142857143 as a fraction?
Answered by Stephen La Rocque and Penny Nom.
Equivalent fractions 2006-09-25
From Orlanda:
How do I write two equivalent decimals for 5.8?
Answered by Penny.
Equivalent fractions 2006-09-20
From Sarah:
I am the aunt of a 5th grader and we were doing math homework and the question was What is the equivalent decimal for .05 and 2.875? I am college education and was completely stumped. Please Help. Thought the question should have been what is the equivalent fraction.
Answered by Penny Nom.
Converting a decimal into a fraction 2006-09-06
From Andres:
How do i convert a decimal into a fraction?
Answered by Penny Nom.
1/x + 1/y 2006-08-11
From Sonya:
what is 1/x+1/y = ? is it equal to 1/x+y or what?
Answered by Penny Nom.
x+ 6 / 3 = x+ 6 / x 2006-08-01
From Lynne:
x+ 6 / 3 = x+ 6 / x
How do I solve this equation?

Answered by Penny Nom.
Converting fractions to decimals 2006-07-25
From Diane:
HOW DO I CONVERT A FRACTION TO A DECIMAL? (E.G,. 8/14 to ? )

And HOW DO I DIVIDE A FRACTION (e.g., 1/14 divided by 3) and then convert that answer to a decimal?

Answered by Penny Nom.
A pole 22 5/6 feet long is broken in two 2006-05-19
From Kenneth:
A pole 22 5/6 feet long is broken in two. One piece is 2 3/4 feet longer than the other. What is the length of each piece?
Answered by Penny Nom.
I want to calculate 0.00353 to the power 1.3 2006-05-14
From Richard:
I want to calculate 0.00353 to the power 1.3 How do I do it?
Answered by Stephen La Rocque.
What is 2/3 divided by 5? 2006-05-11
From KH:
What is 2/3 divided by 5?
Answered by Penny Nom.
A word problem with fractions 2006-05-02
From Stuart:
I have 54 1/4 yards of material. i need to cut as many pieces as possible out of it that are 3 1/12 long. how many pieces will you get and how much is left over?
Answered by Stephen La Rocque.
Adding with mixed fractions 2006-03-30
From Becca:

can anyone help with how we change to get the same denominator with adding whole number.

example:8+7 3/8

6 1/2 + 2 5/6


Answered by Stephen La Rocque.
Adding fractions 2006-03-26
From Barbara:
I know that to subtract 1/4 from 2/3 I must find a common denom. Now the 2/3 becomes 8/12.....i understand the 12, but where does the 8 come from?
Answered by Penny Nom.
Comparing Fractions 2006-03-24
From J.:
how can you compare fractions with like numerators and UNlike denominators, like 2/3 and 2/5, without renaming or using fraction strips?
Answered by Stephen La Rocque and Penny Nom.
x+6/7 = 3/5 2006-03-22
From Cavell:
i couldn't find in the book on how to solve x+6/7 = 3/5 i know its simple but i don't remember how and the book never explains step by set on how to solve problems and i don't have a teacher to ask because i do work through mail so i cant ask questions vary easily
Answered by Stephen La Rocque.
A hat contains between 10 and 25 marbles 2006-03-06
From Kerry:
A hat contains between 10 and 25 marbles. Some marbles are green, and the rest are yellow. Without looking you are to reach into the hat and pull out a marble. The probability of pulling out a green marble is 2/9. How many marbles are in the hat and explain?
Answered by Stephen La Rocque and Penny Nom.
8/11 of 900 2006-03-02
From Cavell:
what the number is 8/11 of 900?
Answered by Penny Nom.
Traveling from Asheville to Indianapolis 2006-02-21
From Grace:
Carol was traveling from Asheville, North Carolina, to Indianapolis, Indiana, by bus. At the halfway point of her trip, in terms of distance, she fell asleep. When she awoke, her distance to Indianapolis was half the distance that she had traveled while she slept. For what fraction of her trip did Carol sleep?
Answered by Penny Nom and Stephen La Rocque.
Equal fractions 2006-02-10
From Dylan:
write three equal ratios

3/4 _________________________


Answered by Penny Nom.
Simplify the variable expression: 3g^2h over 12gh 2006-02-09
From Andrew:
okay the question says simplify the variable expression: 3g2h over 12gh
how would u do this?

Answered by Penny Nom.
Finding the nth term in fractions 2006-01-23
From Zarina:
Each term in this sequence is made by increasing the numerator by 1 and the denominator by 3. here are the first five terms:

1/4, 2/7, 3/10, 4/13, 5/16, ...

(a) write an expression for the nth term:

Answered by Penny Nom.
1/3 of 1/4 2006-01-21
From Nicole:
WHAT FRACTION IS EQUAL TO 1/3 OF 1/4
Answered by Penny Nom.
Convert to decimal form 2006-01-12
From A student:
I would like to know the full answer on how to turn the fraction 3 3/8 into a decimal please.
Answered by Penny Nom.
Solving with fractions 2005-12-14
From A student:

Solve:
1. 3/10k - 1 = 1/2
2. 7 3/4 = 3/8h + 6
3. About how heavy should an object be before you start to measure the object in tons instead of pounds? Explain


Answered by Penny Nom.
5(x-3)/4 =x+1 2005-12-14
From Jennifer:

I have two problems i am stumped on I don't know how to help my daughter.

5(x-3)/4 =x+1
3(x+12)/5 =x +2


Answered by Penny Nom.
2/n +4 = 10- 4/3n 2005-12-11
From Jennifer:

2/n +4 = 10- 4/3n

5(x-3/4) = x+1


Answered by Penny Nom.
How do you simplify a fraction if one of the numbers is negative? 2005-12-05
From Stephanie:

Question: How do you simplify a fraction if one of the numbers is negative?

Ex. -32/40


Answered by Penny Nom.
11 /12 + b = 5/6 2005-12-02
From Jennifer:
11 /12 + b = 5/6
Answered by Penny Nom.
Common fractions to decimals 2005-11-25
From A student:
how do u turn 8 over23 into a decimal and how do u turn 8 over 29 into a decimal
Answered by Penny Nom.
Adding improper fractions 2005-11-25
From Paula:
I would like a simple step by step explanation on how to add improper fractions.
Answered by Penny Nom.
Improper fractions 2005-10-21
From Joe:
Is a whole number an improper fraction. For example, is 4 an improper fraction? Is 4 an improper fraction? Is 4 the simplest form for 20/5? Are 4 and 20/5 equivalent fractions?
Answered by Penny Nom.
Isn't 1/3 written as a decimal .33? 2005-09-21
From Lisa:
Isn't 1/3 written as a decimal .33? My son informs me I'm wrong, but doesn't know the answer
Answered by Chris Fisher and Penny Nom.
Equivalent fractions 2005-09-13
From Cindy:

I am a parent and aunt of a 5th grader.

My nephew has asked me what the equivalent decimal is of 0.4. Does he change it to a fraction or just add a zero either before or after?


Answered by Chris Fisher and Harley Weston.
3A/4 +1=(2A-1)/3 2005-08-21
From Rachael:

I'm doing a practice test question and the answer I keep turning out is different from the answer on the answer sheet. I think I might be forgetting a step. The problem is:

3A/4 +1=(2A-1)/3

The answer is -16. If anyone could explain the steps to me to getting the answer I would be grateful. Whenever I do it I get something completely different.


Answered by Penny Nom.
Stock in a printing company 2005-08-12
From Tracy:
Sam and Mary each owned one-half stock in a printing company. Sam sold 2/3 of his stock to Mary. What fractional part of the printing business does Mary now own?
Answered by Penny Nom.
Converting rational numbers 2005-08-07
From Joe:

I am helping my son with his converting rational
numbers in the form of A/B where A and B are integers and
B not equal to zero

I think I am doing this right but I am not sure so
below are the problems and our answers are beside
them, please let me know if these answerers are correct


Answered by Penny Nom.
Solve for x 2005-07-20
From Ed:
(x-1)/3 - 1 = (x + 2)/2
Answered by Penny.
Understanding fractions 2005-02-11
From C.J.:
I have some questions about the lesson, "Understanding Fractions" by Diane Hanson, Regina Catholic Schools. How successful was the lesson? Are there any changes you would recommend?
Answered by Diane Hanson.
The least common denominator 2004-11-03
From A student:
Write the LCD for each pair of fractions.

13. 1/3,1/5

14. 2/7,1/4

15. 3/4,3/5

Answered by Penny Nom.
Ratios and rates 2004-10-27
From Kenneth:
What is the difference between a ratio and a rate?
Answered by Penny Nom.
Combined operations 2004-08-26
From Louise:
Question 2 1/4 x 1/8 / 1 3/4 x 12 4/9 x 3 =
Answered by Penny Nom.
The horizontal fraction bar 2004-05-27
From Kashia:
What is the name of the bar that separates a numerator and that denominator?
Answered by Penny Nom.
33 3/4 doubled 2004-05-22
From A tailor:
need to know how to add 33 3/4 doubled. How do I come up with the solution? I need to know for measuring garments. Usually I measure on the half meaning on the front of the garment and then double it with the measuring tape but I always get stuck when I need to measure 3/4 such as 66 3/4 doubled and etc. Can you help me figure this out easily.
Answered by Penny Nom.
Mrs. Smith's class 2004-05-12
From Amanda:
Mrs. Smith's class has 35 student. Today there are 2/3 of them absent. How many are absent? present?
Answered by Penny Nom.
Subtracting fractions 2004-05-11
From Filipe:

Question:
_5_ - __7__
6ab 8a


Answered by Penny Nom.
3x squared - 27 / x + 3 2004-05-04
From Stef:
3x squared - 27 / x + 3
Answered by Penny Nom.
A fraction of a fraction as a percent 2004-03-18
From Mary:

I would like to know if you could help me with this problem? Could you take me though the steps
to this math problems.

 3 =(?)% x 3
---       ---
16         4
	  

the question states: Finding what is the percentage of a fractional number and another fractional number.


Answered by Penny Nom.
Fractions 2004-02-13
From A parent:
12/36=?/18=4/?=?/9
Answered by Penny Nom.
Common fractions to decimals 2004-02-12
From A parent:
I'm trying to help my son work with turning fractions into decimals and this seems foreigh to me after 20 years of not being in school. Can you give me a few illustrations and/or examples of how to do that??
Answered by Penny Nom.
A/30 + B/105 = (7A + 2B)/x 2004-02-05
From Jim:
If A/30 + B/105 = (7A + 2B)/x and A, B, and x are integers greater than 1, what must x equal?
Answered by Penny Nom.
3/8 as a decimal 2004-01-13
From A student:
My Math Midterm is tommorow and I am stumped... how do you turn 1 and 3/8 into a decimal... I know the answer is 1.375 but I just copied down what the teacher wrote on the board.
Oh, and what are the three cases of percents?

Answered by Penny Nom.
Laying decorative bricks 2004-01-07
From Duk:
Loren is laying decorative brick along both edges of the 21-meter walkway up to his house. Each brick is 0.26 meters long. He is placing the bricks end to end. How many bricks does he need to do the job?
Answered by Penny Nom.
Fractions 2003-12-31
From Jae:
Replace the question marks with numbers that will make the sentence true.
1.1/3=?/9=?/6
2.?/18=8/12=4/?
3. 3/?=12/?=9/?
4.?/3=?/21=?/7
5. Which problems have more than one possible answer? Why do you think this is so?

Answered by Penny Nom.
Adding fractions 2003-11-16
From Ken:
My name is Ken and I am taking my GED course for my High School and have not been in a class for 35 years. I am doing this for re-training. I am at the part about fractions. Here is an example that I am having trouble with.

1 3/7 + 4 2/3 + 11/21

They have no common denominators. Could you PLEASE help me. If you could send me a step by step explanation it would be greatly appreciated.

Answered by Penny Nom.
1/x + 1/y = 5/12 2003-10-30
From Ben:
1/x +1/y =5/12, what is the sum of x and y?
Answered by Penny Nom.
12 cookies 2003-10-29
From Joel:
there are 12 chocolate, sugar and cinnamon cookies. the probability of choosing a chocolate cookie is 1/3. the probability of choosing a sugar cookie is 1/4 what is the probability of choosing a cinnamon cookie
Answered by Penny Nom.
0.810 and 0.801 2003-10-24
From Ahmed:

could you please tell me what is the greatest decimal fraction formed of the digits 0, 1 , 8?

Is it 0.810 or 0.801, please give me the reason of the answer in details and the references if it possible.


Answered by Penny Nom.
Why percentage? 2003-09-03
From Vicky:
My cousin aske me a question about the use of percentages and I'm not sure how to answer it. He wants to know why when there is a sale in a shop we use percentages and not fractions or decimals.
Answered by Penny Nom.
Is 3/5 CLOSEST to 0, 1/2 or 1? 2003-07-09
From A student:
I would like to know is 3/5 CLOSEST to 0, 1/2 or 1. And can 3/10 go either way as to be closest to 0 or 1/2.
Answered by Penny Nom.
Subtracting rational expressions 2003-05-10
From Simone:
hi, i'm totally lost. i understand that you need to find a lowest common denominator to subtract two fractions (rational expressions) with different denominators. but what if the denominators are "x-1" and "x". is x the common denominator? if so what happens to the "-1"? do you know of any live online help i can get with the following:

3/(x-1) - (1-2x)/x

i've looked through my notes and have no examples that quite match that i can follow to get through it. please help!
Answered by Penny Nom.

A fractional inequality 2003-04-08
From Jessica:
Solve for x. Write in interval notation.

[(x2)-9]/[x-5] >= 0

Answered by Penny Nom.
How many hits? 2003-04-06
From Jack:

My name is Jack. I'm a uncle. Student is in the 5th grade email is above.

If a baseball player at sping training had a good season with the following:

one seventh of his hits were doubles.
12.5% of his hits were home runs.
But didn't have any triples.
How many hits did he have?

Can you give me an explanation of you solved the problem.


Answered by Penny Nom.
A number line 2003-02-27
From Shery:

My seventh grader problem of the month

0__________1______________5____>

This is a arrow, the number should be below

A.Mrs Decker created an arrow representing a number line shown above. She wanted to find points and label them with a heart (G) for Valentine's Day so that the fraction 5/g is less than 1. (be sure to mark the G and not the fraction 5/g). She pondered, "Are there any other locations for G?" Is so help her description the location of all these points. If not why not?


Answered by Penny Nom.
Storyteller figurines 2003-02-10
From A student:
It takes 3/4 of an hour to bake a storyteller figurine. If only one figurine can be baked at a time, how many can be baked in 6 hours?
Answered by Penny Nom.
Rational expressions 2003-01-22
From A student:

// = the main fraction line

1.) a+1/a-1 + a-1/a+1 // a+1/a-1 - a-1/a+1

2.) 2/a(squared)-3a+2 + 2/a(squared)-a-2 // 2/a(squared)-1 + 2/ a(squared)+4a+3


Answered by Penny Nom.
Fractions 2002-12-10
From Jean:
What are some activities to use with advanced students in grade 7 to apply their knowledge of fractions and decimals. I'm looking for a project type of activity.
Answered by Diane Hanson.
What percent is the same as three eighths? 2002-11-21
From Katelin:
What is the percent of three eight's?
Answered by Penny Nom.
Radicals with fractional radicands 2002-11-11
From A student:
how do I simplify radicals with fractional radicands?

ex. 7sqrt(2) + sqrt(50) - 2sqrt(18)

10sqrt(3/5) - 24sqrt(5/3)

3sqrt(2/9) + 1/2sqrt(32) + sqrt(9/8)


Answered by Penny Nom.
At the zoo 2002-10-20
From A student:
The number of adults at the zoo was 2/9 the number of children. there are 700 more children than adults. The number of men was four times the number of women. The number of girls was four times the number of boys. Find the total nymber of men and boys at the zoo.
Answered by Penny Nom.
Rational expressions 2002-10-03
From Ashley:
1/x(squared) + 5/xy
Answered by Penny Nom.
Dividing fractions 2002-09-23
From Angie:
When I was taught to divide a fraction, I was told to multiply the reciprocal. Many times I find I can just divide the fraction, and it saves all the simplifying after.

For example, 4/6 x 9/14 divided by 2/4

I would say 4x9divided by 2=18 over 6x14divided by 4 =21, so the answer is 18/21 but if I multiply the reciprocal, I would end up with 4x9x4 over 6x14x2 which equals 144/168.

So, why are we taught to do it this way? Is it necessary?


Answered by Chris Fisher.
Fractional exponents 2002-09-20
From Jill:
The problem is with fractional exponants:
10 1/3 mult. by 10,000 The 1/3 is an exponant of 10.

Answered by Penny Nom.
1/8 in decimal form 2002-09-04
From A student:
explain how you put 1/8 in a decimal form
Answered by Penny Nom.
Common Denominator 2002-08-26
From Slobodanka:
What is a Common Denominator?
Answered by Penny Nom.
One-fourth of a number is added to one-third of the same number 2002-05-26
From A student:
When one-fourth of a number is added to one-third of the same number, the result is 28. What is the number?
Answered by Penny Nom.
0/4 = ? 2002-04-28
From Danielle:
I am embarrased to be asking this question, but... is it possible to have a fraction with a zero? For example, 0/4. This does not make sense to me and I do not know what it would be representing, other than nothing! Is it proper to express such a fraction?
Answered by Peny Nom.
Solve for x 2002-04-20
From A student:
solve for x.
  c-x      2x+b ----- = --------   a        c 

Answered by Peny Nom.
Common fractions to decimal fractions 2002-04-02
From Natalie:
My questions are: "how can I change 23/60 into a decimal. and give my answer to 2dp."??????????

2nd question:for each of these pairs, which is bigger?

4/7 or 36%____________
3/11or 0.29____________
14/24 or 0.57___________


Answered by Penny Nom.
The reciprocal of a decimal mixed number 2002-03-29
From A student:
How do I find the reciprocal of a decimal mixed number?
Answered by Penny Nom.
Unit fractions 2002-03-23
From Laura:
Write 1/2 as the sum of ten different unit fractions.
Answered by Claude Tardif.
What fraction of the world's motor vehicles are built in Canada? 2002-03-20
From A student:
About 1/4 of the world's motor vehicles is built in Canada or the United States. About 1/5 of the world's motor vehicles are built in the United States. What fraction of the world's motor vehicles are built in Canada?
Answered by Claude Tardif.
Adding algebraic fractions 2002-03-20
From Dolores:
I get totally confused with this problem. I get confused with the getting the lowest terms.

4/x-5 + -2/x -10/x^2-5x


Answered by Penny Nom.
How much olive oil? 2002-03-15
From Annie:
Joe used 1/3 cup of olive oil, which was 3/4 of the amount that he had. How much olive oil did he have!
Answered by Penny Nom.
Mixed numbers, whole numbers and improper fractions which are less than 2. 2002-03-03
From Dawn:
Change each mixed number to a whole number and an improper fraction which is less than 2. 3 1/2 = 2 /
Answered by Penny Nom.
I am a fraction 2002-02-01
From Anthony:
I am a fraction that is greater than 1 but less than 2. The sum of my numerator and denominator is 11. My denominator subtracted from my numerator is 1. What fraction am I?
Answered by Paul Betts.
Adding algebraic fractions 2002-01-23
From Francine:

Hello, I've come up with an answer for the following problem but it's not the same as the book's answer. However, the book has a lot of mistakes in it and I don't know if I'm doing it properly or not. Your help would be soooo appreciated. It's driving me crazy.The problem is:

   
  2             + 3          + 4 
---------  +  ---------  +  ---------- 
(x-1)^3        (x-1)^2       (x-1) 

[(x-1)^3 is (x-1) to power of 3 etc.]


Answered by Penny Nom.
2/3 vs 3/4 2002-01-02
From Julia:
So, here's my question. Some of the problems they've had to do starts off with two fractions, and they have to tell whether the two fractions are:
  1. The first one is greater than the second one
  2. The first one is less than the second one, or
  3. They are equal.

One day my son had the problem of 2/3 vs 3/4. Without drawing out a pie chart, how in the world can a child "eyeball" a problem like this and tell what the correct answer is? I've also tried to teach about making common denominators, but that's going over his head at this point.


Answered by Denis Hanson and Harley Weston.
Fractions to decimals and percents 2001-12-31
From A student:
2/5 , 4/8 , 4/10 , 3/5 , 6/8 , 7/7 (what is the decimal and percent for each)

1/5 , 1/10 , 1/8 , 1/1 (what is the decimal for each)

100 PERCENT, WHAT IS THE DECIMAL?

The ratio of boys to girls in a class is 1:4. What percent of the class are girls?

Answered by Penny Nom.
Reduce 79,537/3,623,420 2001-12-13
From Eliana:
How do you reduce 79,537/3,623,420.
Answered by Claude Tardif.
Fractions in lowest terms 2001-12-05
From Thomas:
how to change decimal to a fraction in the lowest terms .7 .65 .019 .0111 .225 .1225 .625 .25 .025 .26 .875 .5 .75 .750 .33 .09 .44 .15 .12 .90 .044 .2500 .95 .500 .02 .40 .125 .1276 .00009 .45 .125 .1276 .375 .975 .054 .0005 .20 .2 .0001 .85 .80 .9 .0007 9.8 10.17 25.75 48.4 80.35 19.64 276.20 31.042 18.1034
Answered by Penny Nom.
83.3% 2001-12-03
From Ellie:
Please explain how a percentage like 83.3% is changed into a fractions. The answer given is 5/6 and I changed 83.3% to a regular decimal of .833, which would be 833/1000. How do they get 5/6 from that? HELP
Answered by Chris Fisher.
3pi/2 - pi 2001-10-29
From Karen:
I need to simplify this

a) 3pi/2 - pi


Answered by Penny Nom.
An algebraic fraction 2001-10-25
From brandi:
4x - 5
_____ =4
3-7x

Answered by Penny Nom.
1:5 2001-09-27
From Sandy:
A colleague and I have a disagreement about dilutions. I seem to remember that I was taught (although it was a long time ago) that a RATIO was written thusly: 1:5 , and read " one part to five parts", for a total of 6 parts. On the other hand, a fraction is written: 1/5 and is read" one part in five" for a total of 5 parts. My colleague says that the ratio reads "one part in five" and is essentially the same as a fraction. Which is right? Have I been doing dilutions wrong all this time? I teach Animal Nursing and need to know where I can find a consise answer to such questions for making up medicated fluids, etc.
Answered by Penny Nom.
Three fractions 2001-09-10
From Kathleen:
a/bc + d/ef + g/hi = 1

Each of three fractions has a one-digit numerator and a two digit denominator. The three fractions together add up to one. Place the nine digits 1-9 into the fractions to make the equation correct.


Answered by Claude Tardif.
The square root of 21/168 2001-09-10
From A student:
can you show me how to do this square root of 21/168
Answered by Penny Nom.
Cakes and ribbons 2001-07-05
From Jennifer:
  1. A cake recipe calls for 3 1/4 cups of flour. You already put in 1 3/4 cups. How much more should you add?

  2. You have 3/4 yard of ribbon. How many 1/8-yard pieces can you cut the ribbon into?

Answered by Penny Nom.
Dividing fractions 2001-05-09
From Rina:
I just wanted to ask if you could help me in math. See I'm having a test soon and its on Dividing Fractions and I just don't get it. My math teacher says that I'll be just fine but I failed my math quiz. I went to ask eric but they could help me so they told me to go to you. So here I am asking you if you could help me.
Answered by Penny Nom.
Adding and subtracting rational expressions 2001-05-03
From Donna:
Adding and subtracting Rational expressions. I am in grade 10 and I am a student here is an example of the questions:

1/(x+1) - 1/(x-1) = ?


Answered by Penny Nom.
Triangles and fractions 2001-04-27
From Constance:
My name is Constance and I am thirteen years old (I am a student). The question that I am queering about I don't understand why you do ONE HALF x the base x the width WHEN YOU WANT TO FIND the area of a triangle? My second question is if you multiply one half and 10 together why does it come out as 5?
Answered by Penny Nom.
Repeating decimals 2001-04-21
From Sarah:
Hi, I'm working on a project for school. The theory I choose was "When turned into a fraction, a repeating decimal has a denometor that is a multiple of three." I have a couple of questions about this topic. My first question is, have you ever heard of this, and what can you tell me about it? My second question is, when I was testing this theory I came across .999... now, when this is a fraction it is 9/9 which is equal to one. The denometor is a multiple of 3, but it's a whole number. I don't understand how a decimal can be equal to a whole number since a decimal is a piece of a whole number. Please don't just show me a math problem, I don't want to see a math problem. I want to see an explanation of this theory and the decimal .999...
Answered by Penny Nom.
Five fourths 2001-04-08
From Linda:
Can you have 5/4 of something?
Answered by Penny Nom.
Equivalent fractions 2001-04-08
From A student:
which of the following pairs of fractions are equivalent?
questions a-f

(a) 1/5 and 6/30
(b) 4/9 and 16/27
(c) 6/10 and 30/50
(d) 15/20 and 48/64
(e) 2/3 and 33/100
(f) 12/32 and 3/16


please show how u got the answer thank you.

Answered by Penny Nom.
Two integrals 2001-04-03
From Jim:
I'm having trouble with these integrals. Can you help me out?

1)the integral of:

x5 arctan x dx

2)the integral of:

2x5 + 9x4 + 19x3 + 13x2 - 5x - 25
----------------------------------------------- dx
x4 + 4x3 + 5x2


Answered by Claude Tardif.
A sugar cane crop 2001-03-27
From A student:
a sugar cane farmer harvested only 1/4 of his crop. he sold 2/3 of his harvest to the mill. what fraction of the total crop did he sell.

darens recipe for florida fudge calls for 5/8 cup of sugar. if he wants to make only 1/2 of the recipe how much sugar should he use.


Answered by Penny Nom.
Why exactly can't you divide fractions? 2001-03-22
From Dennis:
Why exactly can't you divide fractions?
Answered by Penny Nom.
Some fractions 2001-02-17
From Lady Bird:
Mr. Bob has 3/4 of a bottle of medicine left. He takes the medicine three times a day. Each dose is 1/16 of the bottle. How many days will it take for Mr. Bob to finish the bottle?

My friend gave me a problem to solve and it is so hard. Can you help me solve it? Here it is: If g- 4.8 = 9.66 and e-1/3 = 1/8, what is the sum of g and e?

Mariko's mother used ribbon to wrap 3 birthday gifts. She had 3 ribbons and each was 72 inches long. She used 8/9 of the first ribbon, 2/3 of the second ribbon, and 5/6 of the the third ribbon. How many feet did she use?


Answered by Leeanne Boehm and Penny Nom.
Part of a part 2001-02-11
From Katie:
Hello! I was looking at your Fraction questions and since i am pursuing to be an elementry school teacher i feel i don't understand one of your responses. How would you explain this to an elem, midd, and high school student. Even me myself-in college really does not understand. The question was: When you multiply two positive fractions less than 1, how does the answer compare to the size of the fraction? Why?
Answered by Penny Nom.
What is wrong with these probelms? 2001-02-09
From Cindy:
Hi, I am a planning on becomming a teacher and i am asked to find out what is wrong with these probelms and how i would go about showing a student what is wrong with them!!

Error in patterns:
13/35=1/5; 27/73=2/3; 16/64=1/4

4/5+2/3=6/8; 2/5+3/4=5/9; 7/8+1/3=8/11

2/3*3=6/9; 1/4*6=6/24; 4/5*2=8/10


Answered by Leeanne Boehm and Penny Nom.
Which fraction is greater? 2001-02-01
From Carol:
Hello My name is Carol I am a student teacher! I am asked to aproach these students on their problems. I am unsure of how to explain to them that these are wrong. Can you help? Thanks.

Iris claims that if we have two positive rational numbers, the one with the greatest numerator is the greatest.

Shirly claim that if we have two positive rational numbers, the one with the greatest denominator is the least. Thanks!


Answered by Claude Tardif.
Fractions of fractions 2001-01-30
From Katie:
When you multiply two positive fractions less than 1, how does the answer compare to the size of the fraction? Why?
Answered by Penny Nom.
Multiplication of fractions 2001-01-08
From Angela:
Questions:

1. represent multiplication of fractions via an area model

2. describe why, not just how, to round decimals


Answered by Penny Nom.
Mixed Fractions 2000-12-09
From Tim Finch:
My son is having problems with mixed decimals and mixed fractions.. I am unable to find any information or examples on this subject so I can show him how to do it.. for example how do you make a mixed decimal or fraction out of 7/16
Answered by Penny Nom.
Equivalent fractions 2000-12-03
From A student:
please explain to us how to find the answer to this problem, we need three fractions that name the same amount as 2
Answered by Penny Nom.
A 16 slice pizza 2000-11-30
From A student:
What is 5/8 of a 16 slice pizza?
Answered by Penny Nom.
Dividing fractions 2000-10-18
From Paula:
Why do you have to change the division sign to a multiplication sign and invert the fraction that follows the division sign in order to get the answer to a division problem when you're working with fractions?
Answered by Penny Nom.
Sunflower seeds 2000-10-09
From A parent:
Find the number of 3/4 pound of sunflower seeds that can be filled with 225 pounds of sunflower seeds
Answered by Penny Nom.
Repeating decimals 2000-10-06
From Mary O'Sullivan:
I understand that with repeating decimals (those with a pattern), the number of digits repeated is put into fraction form with the same number of 9's

ex. 0.4444 = 4/9 0.145145145 = 145/999

How can I explain why this is so? Some inquisitive 8th graders are anxious to find out!
Answered by Penny Nom and Walter Whiteley.

Manipulating powers 2000-09-13
From Phil:
21/5 x 42/5 = ?
Answered by Penny Nom.
Subtracting fractions 2000-08-26
From Sarah Stanczyk:
how is this problem solved??

3/ x+1 - 5/x
( 3 divided by x +1 subtracted by 5 divided by x)


Answered by Penny Nom.
Covering 1, 1/2, 1/3,... 2000-08-22
From Wallace:
Consider the points 1, 1/2, 1/3, ... on the real number line. You are given five small bars, all of length p, which are to be placed on the number line such that all points will be covered. What is the minimum value of p that will allow you to do this?
Answered by Penny Nom.
Numerator and denominator 2000-06-18
From Maureen Beard:
What is the origin of the terms numerator and denominater?
Answered by Penny Nom.
Percents with mixed numbers 2000-06-04
From Julie:
How would you find the percent of 33 1/3 out of 90 by using fraction settup!I can do whole numbers but not mixed. This is how to do whole numbers say it was 33 of 90

33 x 90 = 2970
100 x 1 = 100
2970 divided by 100
the answer is 29.7

so how do you do it with a mixed number


Answered by Penny Nom.
Multiplying fractions 2000-05-14
From A student:
75/2 x 1/100 = 3/8

Please show how to came up with the answer?
Answered by Penny Nom.

Why is 1/4 larger than 1/5? 2000-05-02
From Sherry:
I have to do a math project for my teacher education class. The question that I am working on is why is 1/4 bigger than 1/5.
Answered by Paul Betts, Claude Tardif and Penny Nom.
What percent is one fraction of another fraction? 2000-04-29
From A student in grade 4:
What is the method for finding out what percent one fraction is of another fraction. For instance 4/5 is what percent of 1/9?
Answered by Penny Nom.
Adding fractions 2000-04-23
From A grade 6 student:
In adding fractions, how do I rewrite the fractions so that the denominators are equal? The problem is 3/4 + 1/6 =
The other problem is 11/18 + 4/9 =

Answered by Penny Nom.
Why are equivalent fractions equal? 2000-04-05
From An elementary student:
What is the mathematical reason equivalent fractions are equal?
Answered by Penny Nom.
Making fruit salad 2000-03-29
From A grade 5 student:
Miss.Muffet males a fruit salad by adding 31/2 pounds of grapes and 4 1/4 pounds of apples to every 2 1/4 pounds of melon. How many pounds of grapes does she use in 50 pounds of fruit salad?
Answered by Claude Tardif.
I am in lowest terms. 2000-03-21
From Travis Fox:
I am in lowest terms. When you double my numerator and my denominator the result is 10/15.
Answered by Penny Nom.
Making dolls 2000-03-07
From A 6th grader:
How many small dolls can be made from 3 lbs. of dough if each doll uses 7/8 lbs
same question if each doll takes 3/8 lbs.

Answered by Penny Nom.
Fractions in algebra 2000-02-27
From Leslie:
Question:
add (4x+1)/(x-8) + (3x+2)/(x+4) + (49x+4)/(x^2-4x-32) and

solve x - 6/(x-3) = 2x(x-3)

Answered by Penny Nom.
Pi 2000-02-22
From Shelley Collier:
Numbers that can be written in the form p/q where p and q are integers, (q not 0), are known as rational numbers. What you are missing is that p and q must be integers. The fact that Pi is irrational means that you can't have a circle with both the circumference and diameter being integers. In fact you can't even have the circumference and diameter both rational since the quotient of two rationals is again a rational.
Answered by Claude Tardif and Penny Nom.
Number sense 2000-02-17
From A college student:
I am a college student and I am to answer the following question: Describe ways of telling by simple inspection when a fraction has a value close to 0; close to 1/2; and close to 1. I am to imagine that I am a 3rd grader answering this question. Any thoughts please?
Answered by Penny Nom.
The sum of repeating decimals 2000-02-15
From Caitlin Harris:
Express 0.111... + 0.121212... + 0.123123123... as a repeating decimal and its equivalent fraction. Show work. Also, are there any extensions to this problem? In other words, are there any questions that we could ask that may be related to this problem?
Answered by Penny Nom.
Ordering fractions 2000-01-05
From A seventh grader:
I am a middle school student (7th grade) and I was wondering if you could help me with ordering fractions from least to greatest the fractions are 31/12, 2 5/6 and 2 3/4.
Answered by Walter Whiteley.
How many 3/5 are in 3/4? 1999-12-06
From Whiteny:
How many 3/5 are in 3/4?
Answered by Penny Nom.
Repeating decimals 1999-11-22
From Andrew:
Is 1.9 repeating the same as 2?
Answered by Walter Whiteley.
Selecting some players 1999-11-20
From Korbin Brown:
Roger Craig, during his term as team manager of the San Francisco Giants, received a strange communication from the team general manager, Al Rosen. Mr. Rosen told him to select 25 players according to this formula: 1/2 of the team had to be outfielders and infielders 1/4 of the team had to be starting pitchers 1/6 of the team had to be relief pitchers 1/8 of the team had to be catchers Roger was a bit confused by Al's request, yet complied anyway. How did he do it?
Answered by Claude Tardif.
Division/Fractions 1999-11-06
From Azmat Hussain:
Is there a way to explain division by fractions using patterns? For example 3 divided by one half.
Answered by Chris Fisher and Penny Nom.
An equivalent decimal 1999-10-14
From Cheryl Wendlandt:
I have a question in my book that is one and five sixths and you are to put them in an equivalent decimal. I'm sick at home or I would be able to ask my instructor. I would appreciate any help.
Answered by Penny Nom.
Equations with fractions 1999-10-10
From Lori:
I'm a secondary student. How would you figure out a problem such as t-2/t =14/3t - 1/3. The book gave one example but it is really easy. The work YOU have to do is totally different. You don't have to tell me the answer just how to do such problems like these.
Answered by Penny Nom.
A fraction problem 1999-09-23
From TruRed:
Seven ninths divided by a negative 3.Could you pleas answer my question and go step by step.
Answered by Penny Nom.
Fractions, percents and decimals 1999-09-19
From Sue Farr:
My son is in 5th grade, and I can't seem to find any papers that have a simple explanation of how to convert decimals into fractions, percents into decimals, and vice-versa.
Answered by Harley Weston.
Repeating decimals 1999-09-18
From Kavoos Bybordi:
I dont know how to change a reccuring decimal to a fraction please could you tell me the method.
Answered by Penny Nom.
How many went to the game? 1999-09-09
From Lisa Schmidt:
If 3/4 more students went to the game than actually did go to the game, 25% of all students in the entire school would have gone. What fraction of students in the entire school went to the game?
Answered by Harley Weston.
Invert and multiply 1999-08-28
From Debbie Walter:
I have another question please, WHY do we invert and multiply when dividing fractions? I know that's what we do but WHY? What is the reasoning behind it?
Answered by Penny Nom.
Repeating decimals 1999-05-21
From Stan:
Hi, I am in Honors Math, and have confronted everyone, including teachers, about repeating decimals. What interests me is the number 0.9... and 1. Everyone says that since there is no number between 0.9...(repeating) and 1, that 0.9... = 1. However, isn't a repeating number a representation of a number, and not a real number? Let's look at it this way. 0.9 is close to 1. 0.99 is closer. 0.99999999999999 is even closer. so, 0.9... is a representation of it's closeness to 1. it's an active number... I don't understand how 0.9... is equal to 1. Please help me prove that 0.9... does NOT = 1.
Answered by Penny Nom.
Re-Percentaging 1999-05-15
From Doug Guion:
My mind has gone blank. I have a series of percentages which have been establised by the same base number. I need to remove one of the percentages (13%) leaving me with 87%. I need to arifically inflate the remaining percentages to total 100%. Any help would be appreciated.
Answered by Penny Nom.
Turning Fractions into Decimals 1999-01-16
From Lynn:
How do you do it step by step
Answered by Jack LeSage.
Fraction division 1998-11-18
From Brett Darrough:
I am needing specific examples that illustrate division of fractions with In-depth understanding. I am enrolled in a CGI (cognitivly guided instruction) course which is trying to develop a method of teaching fraction division with understanding. Most people invert and multiply to obtain a correct answer. Why? I would like help in understanding this method that includes pictures and algorithms that illustrate reasoning and depend on understanding.
Answered by Jack LeSage and Harley Weston.
Fractions 1998-10-27
From Jessica Braimoh:
I have a problem that I just can't figure out!!

Put into a fraction a) -1.157... b) 0.046... c) 0.032... d) -0.3234... Please show how you got the answer and how you came to it(step by step)
Thanks Jessica Braimoh grade 9 Ontario
Answered by Harley Weston.

Adding Fractions 1998-10-15
From Lindsay:
please help me!!!!
Answered by Harley Weston.
Fractions 1998-10-07
From Nikki Lakevold:
-5 over 9 minus -2 over 3 plus -7 over 6 = ???

This is a fraction question and I really don't know why the back of my textbook says -19 over 18 when I got -23 over 18,, am I wrong???
Answered by Harley Weston.

A Sum or Product of Fractions 1998-10-06
From Emily Robb:
Find a pair of fractions that when they are multiplyed and added the sum /product is the same
Answered by Penny Nom.
Adding Fractions 1998-10-03
From Pam Bailey:
Can you help me simplfy this?

(1/2a + 1/3b) - (1/4a - 1/5b) + (1/6a - 1/7)

thanx
Answered by Harley Weston.

Unit Fractions 1998-09-16
From Murray MacNeil:
Sir, my nephew is having trouble with math,he is in grade 10. this is his question.

Find seven unit fractions whose whole sum is one.

thank you
murray macneil
Answered by Penny Nom.

Rational Numbers 1998-09-05
From Kristin Mckenzie:
Iam a secondary student with a math question I don't know how to do.
This is the question:

Explain whether each decimal number represents a rational number.

(a) 0.16

(b) 0.12

(c) -3.125

(d) -0.27

(e) -0,212 112 111 2 ....

(f) 0. 457 92

If it wouldn't be a problem i would really appreciate it if you sent back the instructions on how to do the question.

My name is Kristin Mckenzie and my return e-mail is lexus1999@hotmail.com

Thank-you for you time
Answered by Jack LeSage and Harley Weston.

Calculus problems 1998-07-13
From Lorraine:
I'm stuck again. Can you help?

This involves integration using the method of partial fractions

the integral of:
7x(to the 5th) - 2x(cubed) + 3 dx
--------------------------------------
x(to the fourth) - 81

Do I have to do long division to reduce the numerator to the fourth power?

the integral of:
4- 16x +21x(squared) + 6x(cubed) - 3x(fourth) dx
----------------------------------------------------
x(cubed)(x - 2)(squared)

Lorraine
Answered by Harley Weston.

Percentages 1998-07-10
From Adam:
Hi,
I would like to know the basics of percentages. It is all rather confusing to me.I have excelled in math but want to be ready next year for more advanced math.

What is Percentages?
Answered by Penny Nom.

Division by a fraction. 1997-11-21
From Roger Marchegiano:
I am interested in a solution to the following extra credit problem for my son in Geometry. We have been unable to produce a satisfactory response:

Show a geometric representation of the division of fractions algorithm, (When you divide fractions you multiply the numerator by the reciprocal of the denominator).
Answered by Penny Nom.

Fractions 1997-10-20
From Rebecca Henry:
When we add fractions, we find a common denominator and add the numerators When we multiply fractions, we simply multiply both numerators and denominators with no regard to commonality.
  1. Why do we not have to find a common denominator when multiplying?

  2. Why do we multiply both numerators and denominators?

Answered by Chris Fisher.
A Question About Pi. 1997-07-08
From Mike Chan:
I have read the section Repeating Decimals in your data base. It mention that 1/17 has at most only 16 repeating digits. But, why does "pi" have an infinite number of digits (and not repeating ).
Answered by Penny Nom.
A proof that e is Irrational. 1997-04-30
From Peter Hall and Jenny:
We have a little mathematical problem... we need some help proving e is an irrational number!

We don't feel very confident in our formulas, so if You have the time to give us a little explanation we would be very grateful!!!
Answered by Doug Farenick and Penny Nom.

The Division Bracket. 1997-04-09
From Judy Riley:
A fellow teacher recently asked if I remembered the exact word for a division bracket (not the symbol with dots, the horizontal line in a fraction, or a solidus). I couldn't. Can you help?
Answered by Walter Whiteley and Harley Weston.
A donkey and his carrots 1997-01-28
From Emily Lind:
There is a donkey who carries carrots. A farmer has 3,000 carrots to carry to the market. The market is 1,000 miles away. The donkey can only carry 1,000 carrots at a time and he eats 1 carrot every mile that he walks but this is only when he is carrying carrots. How many carrots can the farmer get to market by having the donkey carrying them?
Answered by Penny Nom.
Repeating Decimals 1997-01-24
From Grant Reed:
Is there a way to tell that the repeating decimal for 1/17 has no more than 16 repeating digits?
Answered by Penny Nom.
Could you tell me the name for the bar in a division problem? 1996-10-21
From Linda:
Could you tell me the name for the bar in a division problem. Not the line with dots on either side but the line that divides the two numbers? My name is Linda. I am asking for my niece who is in 8th grade.
Answered by Chris Fisher.
algèbre et fractions 2007-02-05
From marion:
Une certaine somme était inscrite sur un livret de caisse d'épargne au 1er janvier. Le 1er mars, on retire le tiers de ce qui était placé au 1er janvier. Le 1er juin, on retire le quart de ce qui était placé au 1er janvier. Il reste sur ce livret une somme égale à 1/7 des sommes retirées plus 1 600 Quel était le montant du livret d'épargne au 1er janvier?
Answered by Claude Tardif.
Le produit des fractions 2007-01-23
From Jasmine:
Au mois de mars, le poids de la brebis augmente d'un huitième. Au mois d'avril, son poids augmente d'un neuvième, au mois de juin d'un onzième. Par quelle fraction faut-il multiplier son ancien poids pour obtenir son nouveau poids?
Answered by Claude Tardif.
les fraction 2006-11-07
From Coullet:
béreger boit les 3/5 d'une bouteille de 3/4 de litre jennifer boit les 4/5 d'une bouteille de 1/2 litre diego boit les 2/5 d'une bouteille de 1 litre, puis le quart de ce qui lui reste
Answered by Claude Tardif.
Fractions 2003-05-10
From Jacques:
Voici la question qu'il pose: 2 x (1/2+1/4) + 3 + 2 x (1/3-1/4). La reponse est 4 2/3. Pourriez m'expliquer etapes par etapes ce que je dois faire.
Answered by Claude Tardif et Diane Hanson.
A/B = C/D 2002-03-06
From Un eleve:
Démontrer que si A sur B et = à C sur D, alors AxD et = à BxC.
Answered by Claude tardin.
Fraction rationnelle 2001-12-11
From Audrey:
réduire en une seule fraction et simplifier.

(4-2x+x2)/(2+x) - (x+2)

Answered by Claude Tardif.
les fractions 1998-02-24
From Colette Huguenin:
Bonjour je révise mes math de tout les secondaire et le livre louer la bibliothèque n'est pas entier voici le genre de problêmes qui me bloque

(5 4/5+1/2)divisé 1/3

je sais que je dois commencer par la parenthèse mais...je fait quoi comme opération avec le 5? dois-je le multiplier ou l'additionner ou.......????? si je pouvais seulement avoir la base des fractions je redébloquerais surement
Answered by Diane Hanson.

 
Page
1/1

 

 


Math Central is supported by the University of Regina and The Pacific Institute for the Mathematical Sciences.

CMS
.

 

Home Resource Room Home Resource Room Quandaries and Queries Mathematics with a Human Face About Math Central Problem of the Month Math Beyond School Outreach Activities Teacher's Bulletin Board Canadian Mathematical Society University of Regina PIMS